Формулы возведение степень в степень: Возведение степени в степень — урок. Алгебра, 7 класс.

Содержание

Возведение в степень эксель

СТЕПЕНЬ (функция СТЕПЕНЬ)

​Смотрите также​SerKol​ вставить, формат сбивается.​И не забывай,​ Нужен ещё один​ использовать вкладку «Формат​ полученная в результате​ и система сама​​ (т.е. со знаком​​Стандартный и самый простой​

Описание

​ целые показатели. А​1​

Синтаксис

​ пример:​

​ и в обычную​Формула​

  • ​ себя максимум диапазон​​2401077,222​Предположим, что вам нужно​

  • ​: Добрый день​​inter​ что основание степени​

Замечание

​ столбец — рядом​ ячеек». В нашем​ математического вычисления корня.​ догадается предложить вам​

Пример

​ «-»).​ вариант – использовать​ вот возведение отрицательного​1​B (число)​ с помощью Excel.​Результат​ чисел до двадцати-тридцати,​=СТЕПЕНЬ(4;5/4)​ вычислить очень маленький​Внимательно прочитал рекомендации,​:​ должно быть положительным​

​ или где-то там. », который​

​ числа в дробную​

​1​C​

​ Ведь для решения​

​2​

​ и то не​Число 4, возведенное в​

​ допуск для детали​

support.office.com>

Как возводить число в отрицательную степень — примеры с описанием в Excel

​ но моей задаче​Puporev​ числом, иначе будет​Далее 3 шага.​ цифру «3» в​ узнать корень в​Как только увидели такую​С использованием мастера функций:​ получается при нажатии​ степень обернется для​1​Преобразование в дробь​ данной задачи можно​7​ более трех-четырех раз.​ степень 5/4.​ механизма или огромное​ это не помогло.​, абсолютно справедливо, степень​ ошибка.​1.​ ячейку «А1», которую​ степени в Excel​ подсказку, сразу жмите​Запускаем мастера функций с​ Shift+6 при английской​

​ вас ошибкой, поскольку​2​Формула​ пользоваться как привычным​-3​ Это не говоря​5,656854249​ расстояние между двумя​Задача — подписать​

​ надо выделить перед​Мариш​В соседнем столбце​ нужно представить в​ то мы не​ на клавишу «Tab». C2​ уж о том,​Со школы всем нам​ галактиками. Для возведения​ размерность на осях​ сменой формата не​: Не могу найти​ в верхней ячейке​ -2 степени.​ используем функцию =КОРЕНЬ().​ Или можете продолжить​

Решение задач в Excel

​ клавиш SHIFT+F3 или​ВАЖНО!​ указанное в начале​4​2​

​ так и удобной​0,002915​ чтобы потом еще​ известно правило о​ числа в степень​

​ графика построенного в​

​ цепляя основное число.​

​ как написать степень​

​ пишешь формулу*: =(первая​

​Последовательность действий следующая:​

​ Вспомним теорию из​

​ писать, вручную вводить​

​ жмем на кнопку​

​Чтобы число было возведено​

​ нашей статьи про​8​7​ для запоминания встроенной​Можно прямо в формуле​ и единицу разделить​ возведении в степень:​ используйте функцию​ Excel.

-C2.​ на результат. Поэтому​ любое число с​СТЕПЕНЬ​ в минус первой​: не получаеться​ Подскажите пожалуйста. Заранее​ в степени.​ по ячейке с​«Корнем n-ой степени от​ в скобках укажите​ формул «fx» (вставить​ степень, необходимо в​ ведь четность –​0,707107​2/5​ несомненный плюс!​Второй вариант – использование​ тем, у кого​ показателем N равно​

​.​

​ степени.​

​inter​

​ спасибо.​

​2.​

​ числом и выбираем​

​ числа а называется​

​ необходимые параметры: два​

​ функцию). Из выпадающего​

​ ячейке поставить знак​

​ это характеристика исключительно​

​7​

​=СТЕПЕНЬ(B2;C2)​

​Перейдем к более сложным​

​ готовой функции «Степень»,​ нет под рукой​ результату перемножения данного​Возвращает результат возведения числа​Может быть кто-нибудь​:​Puporev​В ячейке чуть​ из выскакивающего меню​ число b, n-ая​ числа через точку​ списка «Категория» выбираем​ «=» перед указанием​ ЦЕЛОГО числа. 3​ ниже пишешь: =(вторая​ вкладку «Формат ячеек».​

  1. ​ степень которого равна​ с запятой.​ «Математические», а в​
  2. ​ цифры, которую вы​Автор: Елена Измайлова​49​
  3. ​Воспользовавшись вышеприведенными правилами, вы​ о том, как​ аргумента – число​ мы расскажем, как​ себя N-ное количество​СТЕПЕНЬ(число;степень)​Казанский​

​, что именно?​M128K145​ ячейка старого столбца)​ Если не получилось​ а», то есть:​После этого нажимаете на​ нижнем поле указываем​ хотите возвести.​Часто пользователям необходимо возвести​343​ можете проверить и​ возводить число в​ и показатель. Чтобы​

​ возвести число в​

​ раз. Иными словами,​

​Аргументы функции СТЕПЕНЬ описаны​

​: Не нашел, как​

​Мариш​

​: Код =СТЕПЕНЬ(x;y) где​

​ в степени. ».​ Как правильно сделать​0,377964​ произведено правильно.​ характера, и увидим,​ использованию, достаточно в​ Excel.​ 3 — это​Число​ меню в 2007.​

​Пишем число потом​

​ что возводим в​

​ числа в степени.​

​ верхней панели или​

​«А корень n-ой степени​

​ высчитанное значение 8.​

​ ОК.​

​Мы возвели 8 в​

​ это с помощью​

​-7​

​В конце нашей статьи​

​ что эта задача​

​ любой свободной ячейке​

​Для разрешения задач с​

​ 7, умноженное на​

​    Обязательный.

Основание; может быть​

​ Вот макрос, который​

​ ctrl+1 окно формат​

​ степень, второе -​

​3. Выделяешь оба​

​ жмем комбинацию клавиш​

​ из числа а​

​Последовательность действий проста, а​

​В появившимся диалоговом окне​

​ «квадрат» (т.е. ко​

​ «Экселя»?​

​-7​

​ приведем в форме​

​ очень просто решается​

​ поставить знак «равно»​

​ возведением в степень​

​ себя три раза,​

​ любым вещественным числом.

​ сделает надстрочными два​

​ ячеек\шрифт\надстрочный и потом​

​ степень в которую​

​ новых числа через​

​ CTRL+1.​

​ будет равен возведению​

​ результат пользователь получает​

​ заполняем поля аргументами.​

​ второй степени) и​

​В этой статье мы​

​49​

​ таблицы с формулами​

​ в Excel.​

​ (=), указывающий на​

​ Excel позволяет пользоваться​

​ то есть 343.​Степень​ последних символа выделенного​ пишем степень числа.​ возводят​ шифт, видишь внизу​В появившемся меню выбираем​ к степени этого​ достаточно быстро. В​ К примеру, нам​ получили в ячейке​ попробуем разобраться с​-343​ и результатами несколько​Если кратко, то алгоритм​ начало формулы, и​ одним из двух​ Еще одно правило​    Обязательный. Показатель степени, в​

​ объекта. Объект может​

fb.ru>

Как возвести число к степени в Excel с помощью формулы и оператора

​ всем пасиб​Мариш​ рамочки выделения квадратик,​ вкладку «Число» и​ же числа а​

​ аргументах вместо чисел​ нужно возвести число​ «А2» результат вычисления.​ популярными вопросами пользователей​#ЧИСЛО!​ примеров, как возводить​ вычисления числа с​ ввести вышеприведенные слова.​ вариантов.​ – возведение любой​ которую возводится основание.​ быть любой, у​M128K145​

Как возвести в степень в Excel?

​: я наверное не​ хватаешь его мышкой​ задаем формат для​

  1. ​ на 1/n», то​ могут быть указаны​ «2» в степень​
  2. ​​ и дать инструкцию​#ЧИСЛО!​
  3. ​ число в отрицательную​ дробным показателем следующий.​ Осталось выбрать две​
  4. ​Первое – это использование​ величины в степень​Вместо функции СТЕПЕНЬ для​

​ которого есть свойство​

Вариант №1.
«

​: Для потомков:​ правильно спросила. мне​ — и тянешь​ ячейки «Текстовый». Жмем​ есть:​ ссылки на ячейки.​

​ «3». Тогда в​

  1. ​В Microsoft Office Excel​ по правильному использованию​0,2​ степень, а также​Преобразовать дробный показатель в​ ячейки, которые будут​ формулы со стандартным​
  2. ​ 0 дает единицу,​ возведения в степень​

​ Characters — ячейка,​Например число 2​ не нужно высчитать.​ вниз на сколько​ ОК.​

​n√a = a1/n.​

Вариант №2. С использованием функции

​Чтобы извлечь корень с​ первое поле вводим​ есть удобная функция​ системы. MS Office​0,2​ несколько примеров с​

​ правильную или неправильную​

​ участвовать в операции​

​ знаком «крышечка». Введите​

  1. ​ а возведение отрицательной​ можно использовать оператор​ автофигура,​
  2. ​ в 3 степени​ мне надо чтобы​ надо строк. Формулы​В ячейке A1 вводим​Из этого следует чтобы​ помощью формул Microsoft​ «2», а во​
  3. ​ «СТЕПЕНЬ», которую вы​ Excel позволяет выполнять​0,04​ оперированием дробными числами​
  4. ​ дробь. 2.​

    ​подпись оси графика​

    1. ​Toxa33rus​ выглядело как число​ тогда расставятся автоматом.​ рядом с числом​ вычислить математическую формулу​ Excel, воспользуемся несколько​ второе — «3».​ можете активизировать для​ ряд математических функций:​0,008​ и степенями.​Возвести наше число в​ числа вручную), и​
    2. ​ листа следующие данные:​ результат обычного возведения​Скопируйте образец данных из​и т.д. Sub​: Формат ячейки должен​ и вверху степень.​_______________________________​ «3» число «-2»​ корня в n-ой​
    3. ​ иным, но весьма​Нажимаем кнопку «ОК» и​ осуществления простых и​ от самых простых​0,447214​Проверьте на рабочем листе​ числитель полученной преобразованной​ нажать на клавишу​B​ в степень, если​ следующей таблицы и​

    ​ bb() Dim x​ быть текстовый. С​Puporev​* — если​

    ​ и выделяем его.​

    1. ​ степени например:​ удобным способом вызова​ получаем в ячейке,​ сложных математических расчетов.​ до сложнейших. Это​2,236068​ книги Excel следующие​ дроби.
    2. ​ Enter. Посмотрим на​C​ она четная, и​ вставьте их в​ Set x =​ общим и числовым​: Формат ячейки -​ не умеешь писать​Снова вызываем формат ячеек​5√32 = 2​
    3. ​ функций:​ в которую вводили​Функция выглядит следующим образом:​ универсальное программное обеспечение​

    ​0,4​ примеры. Чтобы все​Из полученного в предыдущем​ нескольких простых примерах.​Формула​ такой же результат​

    Корень в степени в Excel

    ​ ячейку A1 нового​ Selection x.Characters(x.Characters.Count -​ точно не прокатывает​ > Шрифт ->​ формулы в экселе,​ (например, комбинацией горячих​

    1. ​В Excel следует записывать​Перейдите по закладке «Формулы».​ формулу, необходимое нам​=СТЕПЕНЬ(число;степень)​ рассчитано на все​0,4​ заработало корректно, вам​
    2. ​ пункте числа вычислить​B​Результат​ со знаком «минус»,​ листа Excel. Чтобы​ 1, 2).Font.Superscript =​inter​ Надстрочный индекс​ нажми на кнопку​ клавиш CTRL+1) и​ через такую формулу:​ В разделе инструментов​ значение. C2​ число в отрицательную​ а затем —​ ответ, но это​ на моем скрине​ что б было​Юрик​ отмечаем галочкой опцию​
    3. ​Или через такую функцию:​ списка указываем на​ Программа подсчитала все​Первая цифра – значение​Число «1» в любой​
    4. ​-0,4​ число, и номер​ этапе.​7​
    5. ​343​ степень. Для этого​ клавишу ВВОД. При​ для меня китайская​ в общем сделано.​ обычным шрифтом, а​: Вот, пункт 2​ «надстрочный». И жмем​ =СТЕПЕНЬ(32;1/5)​
    6. ​ опцию «КОРЕНЬ».​ верно и выдала​

    ​ «число». Это основание​ степени будет оставаться​-0,4​ строки, содержащей показатель.​Согласитесь, что даже при​3​

    exceltable.com>

    Как возвести в степень весь столбик с числами в excel?

    ​Таким же образом можно​​ нужно возвести обычным​ необходимости измените ширину​ грамота. Если можно,​Delphin_KKC​ часть надсрочным индексом.​
    ​ у Аллы лишний.​
    ​ ОК.​
    ​В аргументах формулы и​Введите аргумент функции по​ вам результат. C$3».​ подобные вычисления могут​3​
    ​ — отрицательную, дробную.​ а потом единицу​Формула​Еще раз спасибо.​ общий прокатит ибо​

    Надстрочный индекс в Excel (было: степень числа в Excel)

    ​ можно просто вставить​​ столбец А, и​Пользоваться возможностями Excel просто​ вместо числа.​ было найти корень​ предлагаем еще один​

    ​ введение любого вещественного​​ «0».​

    ​#ЧИСЛО!​​Число / Степень​ занять немало времени.​7​ Выполним следующие действия​ поделить на результат.​Описание​

    ​SerKol​​ эксель воспринимает сие​ символ (Вставка /​ тебе все числа​ и удобно. С​Часто вам важно, чтобы​ из цифры «25»,​

    ​ простой вариант.​​ числа.​Любое число, возведенное в​Обратите внимание, что положительные​

    ​1​​ Хорошо, что табличному​-3​ и ответим на​Из этих правил становится​Результат​, выложите книгу с​ как текст. А​ Символ… — в​ в нём нужно​ ними вы экономите​ число в степени​

    ​ поэтому вводим его​​Ввод функции вручную:​Вторая цифра – значение​ нулевую степень, равняется​
    ​ числа (даже нецелые)​2​

    ​ процессору Excel без​​=СТЕПЕНЬ(B3;C3)​​ вопрос о том,​​ понятно, что выполнение​=СТЕПЕНЬ(5;2)​ примерными данными и​ если пробела не​

    ​ группе «латиница-1»).​ возвести в степень​

    ​ время на осуществлении​​ корректно отображалось при​​ в строку. После​​В строке формул ставим​

    ​ «степень». Это показатель,​​ единице.​
    ​ без проблем вычисляются​3​ разницы, какое число​0,002915​ как возвести число​

    ​ реальных задач с​​Число 5 в квадрате.​
    ​ поясните, что нужно​ ставить (что в​

    ​Puporev​​ 2/3. Тогда в​ математических подсчетов и​ распечатывании и красиво​ введения числа просто​

    ​ знак «=» и​​ в который мы​​Любое значение «А» в​​ при любых показателях.​0,5​ и в какую​

    ​Как видим, нет ничего​​ в отрицательную степень.​ оперированием большими величинами​25​ сделать.Выделить — Главная​ данном случае предпочтительнее)​: Например 2 пробел​ ячейке B1 пишешь​ поисках необходимых формул.​ выглядело в таблице.​ нажимаем на кнопку​ начинаем вводить название​

    ​ возводим первую цифру.​​ степени «1» будет​
    ​ Не возникает проблем​-0,5​ степень возводить. (2/3), и​

    ​Алла горская, педагог-психолог​​ Как в Excel​ «ОК». В ячейке​ функции. Обычно достаточно​Значения обоих параметров могут​ равняться «А».​ и с возведением​1​ решить на рабочем​ как возводить число​B​ средств. Вручную получится​​Число 98,6, возведенное в​​ Шрифт — здесь​ не катит, нужно​ потом формат надстрочный​ «протягиваешь» эту формулу​: У тя есть​ написать число в​

    ​ будет отражена цифра,​​ написать «сте» -​ быть меньше нуля​Примеры в Excel:​ любых чисел в​1​ листе Excel следующий​
    ​ в отрицательную степень​

    ​C​​ перемножить на самого​ степень 3,2.​ отметить «надстрочный»​ текстовый.​Не получается сюда​ до конца столбца.​ столбец с числами.​

    CyberForum.ru>

    ​ степени? Здесь необходимо​

    • Как в эксель степень поставить
    • В эксель возвести в степень
    • В excel знак степени
    • В эксель порядковый номер
    • В excel возведение в квадрат
    • Включить макросы в эксель
    • В эксель межстрочный интервал
    • Поиск по двум критериям в эксель
    • Эксель руководство пользователя
    • Нумерация в эксель автоматически
    • Знак доллара в эксель
    • Замена в эксель

    Возведение степень в экселе функция.

    Также в рамках формулы можно одновременно использовать, как адреса ячеек, так и обычные числа.

    Возведения в степень при помощи формулы

    Кроме этого в Экселе есть формула «СТЕПЕНЬ», с помощью которой можно возвести в степень содержимое ячейки или просто число. Для того чтобы воспользоваться данной формулой установите курсор там, где должен быть результат возведения в степень, и введите формулу «=СТЕПЕНЬ(B2;B4)». В данной формуле B2 – адрес ячейки, значение которой нужно возвести в степень, а B4 – это адрес ячейки, содержащей степень. Обратите внимание, формула начинается со знака «=» и не содержит пробелов, а для разделения адресов ячеек используется запятая.

    При необходимости в формуле «СТЕПЕНЬ» можно использовать как адреса ячеек, так и обычные числа.

    Написание числа в степени

    Если вам нужно просто написать число со степенью, то это также можно сделать в Экселе. Для этого сначала нужно изменить формат ячейки, в которой будет число со степенью, на текстовый. Поэтому кликаем правой кнопкой мышки по нужной ячейке и выбираем «Формат ячеек».

    В открывшемся окне выбираем формат ячейки «Текстовый» и закрываем окно кнопкой «Ok».

    После этого вводим в эту же ячейку число и степень, в которую нужно возвести это число. Дальше выделяем степень числа и кликаем по выделенному правой кнопкой мышки. В появившемся контекстном меню снова выбираем «Формат ячеек».

    После этого перед вами появится окно с настройками шрифтов. Здесь нужно отметить функцию «Надстрочный» и закрыть окно кнопкой «Ok».

    В результате вы должны получить число со степенью, как на скриншоте внизу.

    Для того чтобы скрыть зеленую пометку рядом с числом, нужно открыть предупреждение и выбрать вариант «Пропустить эту ошибку».

    Часто пользователям необходимо возвести число в степень. Как правильно сделать это с помощью «Экселя»?

    В этой статье мы попробуем разобраться с популярными вопросами пользователей и дать инструкцию по правильному использованию системы. ».

Мы возвели 8 в «квадрат» (т.е. ко второй степени) и получили в ячейке «А2» результат вычисления.



Вариант №2. С использованием функции

В Microsoft Office Excel есть удобная функция «СТЕПЕНЬ», которую вы можете активизировать для осуществления простых и сложных математических расчетов.

Функция выглядит следующим образом:

СТЕПЕНЬ(число;степень)

ВНИМАНИЕ!

  1. Цифры для этой формулы указываются без пробелов и других знаков.
  2. Первая цифра – значение «число». Это основание (т.е. цифра, которую мы возводим). Microsoft Office Excel допускает введение любого вещественного числа.
  3. Вторая цифра – значение «степень». Это показатель, в который мы возводим первую цифру.
  4. Значения обоих параметров могут быть меньше нуля (т.е. со знаком «-»).

Формула возведения в степень в Excel

Примеры использования функции СТЕПЕНЬ().

С использованием мастера функций:

Если лишние клики вы считаете сомнительным удовольствием, предлагаем еще один простой вариант. (1/n)- где a-число; n-степень:

Или через такую функцию: =СТЕПЕНЬ(32;1/5)

В аргументах формулы и функции можно указывать ссылки на ячейки вместо числа.

Как в Excel написать число в степени?

Часто вам важно, чтобы число в степени корректно отображалось при распечатывании и красиво выглядело в таблице. Как в Excel написать число в степени? Здесь необходимо использовать вкладку «Формат ячеек». В нашем примере мы записали цифру «3» в ячейку «А1», которую нужно представить в -2 степени.

Последовательность действий следующая:


Пользоваться возможностями Excel просто и удобно. С ними вы экономите время на осуществлении математических подсчетов и поисках необходимых формул.

Excel — мощный табличный процессор, разработанный для решения определенных вопросов. С его помощью можно отображать данные в виде таблицы, производить вычисления, ориентируясь на поставленные цели и задачи, представлять полученные результаты в виде диаграмм. С помощью Excel достаточно просто выполнять такие действия, как:

  1. Математические расчеты (сложение, вычитание, корень числа, возведение в квадрат, куб и другую степень в Экселе). 2», что означает 4 возвели в квадрат. Для работы с числовыми величинами необходимо, чтобы в ячейках был установлен формат «числовой». Его можно выбрать в диалоговом окне «Формат ячейки».

    Использование мастера функций

    Если вы помните синтаксис нужной функции, то ввести ее можно в выбранной ячейке, предварительно начав со знака равенства. В том случае, когда количество аргументов, порядок и правила их записи вызывают затруднения, целесообразно применить , существующий в Excel. Это позволит правильно ввести имя функции и ее аргументы. Порядок действий приведен далее:

    1. Для это помещаем текстовый курсор в ячейку, в которой будет находиться итоговое значение (это можно сделать двойным щелчком мыши).
    2. Выбираем «Вставка», «Функция» (или одновременное нажатие клавиш Ctrl+F2).
    3. На вкладке «Функции» в категориях находим «Математические», потом в списке с прокруткой мы выделяем СТЕПЕНЬ (POWER).
    4. «Далее».
    5. В обозначенных полях вводим основание и степень, например, нам нужно возвести 7 в квадрат, значит, основание равно 7, степень равна 2. » можно возводить не только обычные числа, но и данные, содержащиеся в определенном диапазоне листа.

      Возведем в шестую степень содержимое ячейки A2.


      Как видим, все значения нужного интервала были возведены в указанную степень.

      Данный способ максимально прост и удобен, и поэтому так популярен у пользователей. Именно он применяется в подавляющем большинстве случаев вычислений.

      Способ 2: применение функции

      В Экселе имеется также специальная функция для проведения данного расчета. Она так и называется – СТЕПЕНЬ . Её синтаксис выглядит следующим образом:

      СТЕПЕНЬ(число;степень)

      Рассмотрим её применение на конкретном примере.


      Вслед за этим результат вычисления данной функции выводится в место, которое было выделено ещё в первом шаге описываемых действий.

      Кроме того, окно аргументов можно вызвать, перейдя во вкладку «Формулы» . На ленте следует нажать кнопку «Математические» , расположенную в блоке инструментов «Библиотека функций» . В открывшемся списке доступных элементов нужно выбрать «СТЕПЕНЬ» . После этого запустится окно аргументов этой функции.

      Пользователи, которые имеют определенный опыт, могут не вызывать Мастер функций , а просто вводить формулу в ячейку после знака «=» , согласно её синтаксису.

      Данный способ более сложный, чем предыдущий. Его применение может быть обосновано, если расчет нужно произвести в границах составной функции, состоящей из нескольких операторов.

      Способ 3: возведение в степень через корень

      Конечно, данный способ не совсем обычный, но к нему тоже можно прибегнуть, если нужно возвести число в степень 0,5. Разберем этот случай на конкретном примере.

      Нам нужно возвести 9 в степень 0,5 или по-другому — ½.


      Но, конечно, к данному способу расчета прибегают довольно редко, используя более известные и интуитивно понятные варианты вычислений.

      Способ 4: запись числа со степенью в ячейке

      Этот способ не предусматривает проведения вычислений по возведению. » . В отдельных случаях можно применить функцию СТЕПЕНЬ . Если вам нужно возвести число в степень 0,5, то существует возможность воспользоваться функцией КОРЕНЬ . Если же пользователь хочет визуально отобразить степенное выражение без вычислительных действий, то тут на помощь придет форматирование.

      Одним из наиболее частых математических действий, применяемых в инженерных и других вычислениях, является возведение числа во вторую степень, которую по-другому называют квадратной. Например, данным способом рассчитывается площадь объекта или фигуры. К сожалению, в программе Excel нет отдельного инструмента, который возводил бы заданное число именно в квадрат. Тем не менее, эту операцию можно выполнить, использовав те же инструменты, которые применяются для возведения в любую другую степень. Давайте выясним, как их следует использовать для вычисления квадрата от заданного числа.

      Как известно, квадрат числа вычисляется его умножением на самого себя. Данные принципы, естественно, лежат в основе вычисления указанного показателя и в Excel. » . При этом, в качестве объекта, который будет возведен в квадрат, можно использовать число или ссылку на ячейку, где данное числовое значение расположено.

      Общий вид формулы для возведения в квадрат следующий:

      В ней вместо «n» нужно подставить конкретное число, которое следует возвести в квадрат.

      Посмотрим, как это работает на конкретных примерах. Для начала возведем в квадрат число, которое будет составной частью формулы.


      Теперь давайте посмотрим, как возвести в квадрат значение, которое расположено в другой ячейке.


      Способ 2: использование функции СТЕПЕНЬ

      Также для возведения числа в квадрат можно использовать встроенную функцию Excel СТЕПЕНЬ . Данный оператор входит в категорию математических функций и его задачей является возведение определенного числового значения в указанную степень. Синтаксис у функции следующий:

      СТЕПЕНЬ(число;степень)

      Аргумент «Число» может представлять собой конкретное число или ссылку на элемент листа, где оно расположено.

      Аргумент «Степень» указывает на степень, в которую нужно возвести число. Так как перед нами поставлен вопрос возведения в квадрат, то в нашем случае данный аргумент будет равен 2 .

      Теперь посмотрим на конкретном примере, как производится возведение в квадрат с помощью оператора СТЕПЕНЬ .


      Также для решения поставленной задачи вместо числа в виде аргумента можно использовать ссылку на ячейку, в которой оно расположено.


      Сформулируйте правило возведения в степень произведения. Степень и ее свойства. Определение степени. Возведение в дробную степень

      Тема урока: Возведение в степень произведения, частного и степени

      Тип урока: Урок обобщения и систематизации знаний

      Формируемые результаты:

        Предметные. Закрепить навыки применения свойств степени с натуральным показателем

        Личностные. Формировать умение планировать свои действия в соответствии с учебным заданием

        Метапредметные. Развивать понимание сущности алгебраических предписаний и умение действовать в соответствии с предложенным алгоритмом

      Планируемые результаты: Учащиеся научится применять свойства степени с натуральным показателем для вычисления значения выражений и преобразование выражений, содержащих степени.

      Оборудование: карточки, мультимедийный проектор, сигнальные карточки для рефлексии.

      Организационная структура урока:

      1 . Организационный момент.

      Здравствуйте, дорогие ребята! Я очень рада вас видеть. Начнем урок математики

      Какие трудности были при выполнении д/з?

      Рефлексия.

      Перед каждым учеником лежат кружки трёх цветов: красный, зеленый, синий.

      Расскажите мне о своём настроении с помощью цветных кружочков (красный – радостное, я уверен, что на уроке узнаю много нового, уверен в своих знаниях.

      Зелёный – спокойное; я уверен в своих знаниях.

      Синий – тревожное; я не уверен в себе).

      Я немного подниму вам настроение словами Пуассона: «Жизнь украшается двумя вещами: занятием математикой и её преподаванием».

      Давайте украшать нашу жизнь!

      2. Сообщение темы и цели урока.

        Сегодня мы продолжим изучение темы: «Возведение в степень произведения частного и степени»,

        закрепим все изученные действия со степенями,

        будем учиться рассуждать, логически мыслить и доказывать свою точку зрения.

      3. Блиц-опрос по правилам темы.

        Как перемножить степени с одинаковыми основаниями? Приведите примеры.

        Как поделить степени с одинаковыми основаниями?

        Чему равна степень числа а, не равного 0, с нулевым показателем?

        Как возвести в степень произведение?

        Как возвести степень в степень?

      4. Устный счет.

      Кому принадлежат эти слова?

      «Среди всех наук, открывающих человеку путь к познанию законов природы, самая могущественная, самая великая наука – математика».

      /Софья Васильевна Ковалевская/

      Первая женщина – ученый-математик.

      Вы узнаете, выполнив задания устного счета.

      К – Чему равна сторона квадрата, если его площадь равна 49см 2 . (7см)

      О – Квадрат какого числа равен ? ()

      В – х 3 х 4 (х 7 )

      А – х 6 : х 2 (х 4 )

      Л – (х 3 ) 3 (х 9 )

      Е —
      (m 3 )

      В —
      (m 8 )

      С —
      (m 10 )

      К – (- 2) 3 (-8)

      А — — 2 2 (-4)

      Я — 2 0 (1)

      5. Закрепление изученного.

      Мы повторили правила возведения произведения в степень и степени в степень.

      Теперь закрепим на практических заданиях.

      Несколько человек займутся исследованием. (Слайд)

      Работа в парах.

      1) Докажите, что квадраты противоположных чисел равны.

      2) Докажите, что кубы противоположных чисел противоположны.

      3) Как изменится площадь квадрата, если его сторону увеличить в 2 раза; в 3 раза; в 10 раз; в n раз?

      4) Как изменится объём куба, если его ребро увеличить в 2 раза; в 3 раза; в 10 раз; в n раз?

      6. Рефлексия: покажите мне своё настроение.

      7. Физминутка: «Согласен – не согласен»

      Качните головой, если согласны со мной или нет.

      1) (у 2 ) 3 = у 5 (нет)

      2) (-3) 3 = -27 (да)

      3) (-х) 2 = -х 2 (нет)

      4) График функции у = 1,3х проходит через начало координат. (да)

      8.

      3 · () 2 – 0,5 2

      а) -1; б) — 1; в) -1; г) 1

      2) Упростите выражение:

      а) m 10 ; б)m 4 ; в) m 2 ; г) m 8 .

      3) Вычислите:

      А) 3; б) 9; в) : г)

      4) Какое выражение надо подставить вместо (*), чтобы получилось тождество:

      Х 8 : (*) = х 4

      А) х 4 ; б) х 2 ; в) х 8 ; г) х 12

      Проверка теста по слайду:

      9. Поиграем «Найди ошибку!»

      1) а 15 : а 3 = а 5

      2) –z · z 5 · z 0 = — z 6 — верно

      3)
      =

      4)(у 4 у) 2 = у 10 — верно

      Выпишите неверные задания и решите верно.

      10. Итог урока.

      Чему научились на уроке?

      11. Д/з

      № 458, 457 (слайд)

      Доклады о С.В. Ковалевской.

      12. Рефлексия.

      Покажите, с какими чувствами вы уходите с урока?

      Слайд: Удачи!

      ФИ:

      Самостоятельная работа. (тест)

      1) Найдите значение выражения:

      3· () 2 – 0,5 2

      а) -1; б) — 1; в) -1; г) 1

      2) Упростите выражение:

      а) m 10 ; б)m 4 ; в) m 2 ; г) m 8 .

      3) Вычислите:

      а) 3; б) 9; в) : г)

      4) Какое выражение надо подставить вместо (*), чтобы получилось тождество:

      х 8 : (*) = х 4

      а) х 4 ; б) х 2 ; в) х 8 ; г) х 12

      Оценка:

      Самостоятельная работа. (тест)

      1) Найдите значение выражения:

      3· () 2 – 0,5 2

      а) -1; б) — 1; в) -1; г) 1

      2) Упростите выражение:

      Возведение в степень – операция, тесно связанная с умножением, это операция – результат многократного умножения какого-либо числа на само себя. 3 = 8 .

      Примеры для решения:

      Возведение в степень презентация

      Презентация по возведению в степень, рассчитанную на семиклассников. Презентация может разъяснить некоторые непонятные моменты, но, вероятно, таких моментов не будет благодаря нашей статье.

      Итог

      Мы рассмотрели лишь верхушку айсберга, чтобы понять математику лучше — записывайтесь на наш курс: Ускоряем устный счет — НЕ ментальная арифметика.

      Из курса вы не просто узнаете десятки приемов для упрощенного и быстрого умножения, сложения, умножения, деления, высчитывания процентов, но и отработаете их в специальных заданиях и развивающих играх! Устный счет тоже требует много внимания и концентрации, которые активно тренируются при решении интересных задач.

      Напоминаем, что в данном уроке разбираются свойства степеней с натуральными показателями и нулём. Степени с рациональными показателями и их свойства будут рассмотрены в уроках для 8 классов.

      Степень с натуральным показателем обладает несколькими важными свойствами, которые позволяют упрощать вычисления в примерах со степенями.

      Свойство № 1


      Произведение степеней

      Запомните!

      При умножении степеней с одинаковыми основаниями основание остаётся без изменений, а показатели степеней складываются.

      a m · a n = a m + n , где «a » — любое число, а «m », «n » — любые натуральные числа.

      Данное свойство степеней также действует на произведение трёх и более степеней.

      • Упростить выражение.
        b · b 2 · b 3 · b 4 · b 5 = b 1 + 2 + 3 + 4 + 5 = b 15
      • Представить в виде степени.
        6 15 · 36 = 6 15 · 6 2 = 6 15 · 6 2 = 6 17
      • Представить в виде степени.
        (0,8) 3 · (0,8) 12 = (0,8) 3 + 12 = (0,8) 15

      Важно!

      Обратите внимание, что в указанном свойстве речь шла только об умножении степеней с одинаковыми основаниями . Оно не относится к их сложению.

      Нельзя заменять сумму (3 3 + 3 2) на 3 5 . Это понятно, если
      посчитать (3 3 + 3 2) = (27 + 9) = 36 , а 3 5 = 243

      Свойство № 2


      Частное степеней

      Запомните!

      При делении степеней с одинаковыми основаниями основание остаётся без изменений, а из показателя степени делимого вычитают показатель степени делителя.

      = 11 3 − 2 · 4 2 − 1 = 11 · 4 = 44

    6. Пример. Решить уравнение. Используем свойство частного степеней.
      3 8: t = 3 4

      T = 3 8 − 4

      Ответ: t = 3 4 = 81
    7. Пользуясь свойствами № 1 и № 2, можно легко упрощать выражения и производить вычисления.

      • Пример. Упростить выражение.
        4 5m + 6 · 4 m + 2: 4 4m + 3 = 4 5m + 6 + m + 2: 4 4m + 3 = 4 6m + 8 − 4m − 3 = 4 2m + 5
      • Пример. Найти значение выражения, используя свойства степени.
        = = =2 9 + 2
        2 5
        =2 11
        2 5
        = 2 11 − 5 = 2 6 = 64

        Важно!

        Обратите внимание, что в свойстве 2 речь шла только о делении степеней с одинаковыми основаниями.

        Нельзя заменять разность (4 3 −4 2) на 4 1 . Это понятно, если посчитать (4 3 −4 2) = (64 − 16) = 48 , а 4 1 = 4

        Будьте внимательны!

        Свойство № 3


        Возведение степени в степень

        Запомните!

        При возведении степени в степень основание степени остаётся без изменения, а показатели степеней перемножаются.

        (a n) m = a n · m , где «a » — любое число, а «m », «n » — любые натуральные числа.


        Свойства 4


        Степень произведения

        Запомните!

        При возведении в степень произведения каждый из множителей возводится в степень. Затем полученные результаты перемножаются.

        (a · b) n = a n · b n , где «a », «b » — любые рациональные числа; «n » — любое натуральное число.

        • Пример 1.
          (6 · a 2 · b 3 · c) 2 = 6 2 · a 2 · 2 · b 3 · 2 · с 1 · 2 = 36 a 4 · b 6 · с 2
        • Пример 2.
          (−x 2 · y) 6 = ((−1) 6 · x 2 · 6 · y 1 · 6) = x 12 · y 6

        Важно!

        Обратите внимание, что свойство № 4, как и другие свойства степеней, применяют и в обратном порядке.

        (a n · b n)= (a · b) n

        То есть, чтобы перемножить степени с одинаковыми показателями можно перемножить основания, а показатель степени оставить неизменным.

        • Пример. Вычислить.
          2 4 · 5 4 = (2 · 5) 4 = 10 4 = 10 000
        • Пример. Вычислить.
          0,5 16 · 2 16 = (0,5 · 2) 16 = 1

        В более сложных примерах могут встретиться случаи, когда умножение и деление надо выполнить над степенями с разными основаниями и разными показателями. В этом случае советуем поступать следующим образом.

        Например, 4 5 · 3 2 = 4 3 · 4 2 · 3 2 = 4 3 · (4 · 3) 2 = 64 · 12 2 = 64 · 144 = 9216

        Пример возведения в степень десятичной дроби.

        4 21 · (−0,25) 20 = 4 · 4 20 · (−0,25) 20 = 4 · (4 · (−0,25)) 20 = 4 · (−1) 20 = 4 · 1 = 4

        Свойства 5


        Степень частного (дроби)

        Запомните!

        Чтобы возвести в степень частное, можно возвести в эту степень отдельно делимое и делитель, и первый результат разделить на второй.

        (a: b) n = a n: b n , где «a », «b » — любые рациональные числа, b ≠ 0, n — любое натуральное число.

        • Пример. Представить выражение в виде частного степеней.
          (5: 3) 12 = 5 12: 3 12

        Напоминаем, что частное можно представить в виде дроби. Поэтому на теме возведение дроби в степень мы остановимся более подробно на следующей странице.

      Возведение в 4 степень. Формулы степеней и корней

      Калькулятор помогает быстро возвести число в степень онлайн. Основанием степени могут быть любые числа (как целые, так и вещественные). Показатель степени также может быть целым или вещественным, и также как положительным, так и отрицательным. Следует помнить, что для отрицательных чисел возведение в нецелую степень не определено и потому калькулятор сообщит об ошибке в случае, если вы всё же попытаетесь это выполнить.

      Калькулятор степеней

      Возвести в степень

      Возведений в степень: 28399

      Что такое натуральная степень числа?

      Число p называют n -ой степенью числа a , если p равно числу a , умноженному само на себя n раз: p = a n = a·…·a
      n — называется показателем степени , а число a — основанием степени .

      Как возвести число в натуральную степень?

      Чтобы понять, как возводить различные числа в натуральные степени, рассмотрим несколько примеров:

      Пример 1 . Возвести число три в четвёртую степень. То есть необходимо вычислить 3 4
      Решение : как было сказано выше, 3 4 = 3·3·3·3 = 81 .
      Ответ : 3 4 = 81 .

      Пример 2 . Возвести число пять в пятую степень. То есть необходимо вычислить 5 5
      Решение : аналогично, 5 5 = 5·5·5·5·5 = 3125 .
      Ответ : 5 5 = 3125 .

      Таким образом, чтобы возвести число в натуральную степень, достаточно всего лишь умножить его само на себя n раз.

      Что такое отрицательная степень числа?

      Отрицательная степень -n числа a — это единица, поделённая на a в степени n: a -n = .

      При этом отрицательная степень существует только для отличных от нуля чисел, так как в противном случае происходило бы деление на ноль.

      Как возвести число в целую отрицательную степень?

      Чтобы возвести отличное от нуля число в отрицательную степень, нужно вычислить значение этого числа в той же положительной степени и разделить единицу на полученный результат.

      Пример 1 . Возвести число два в минус четвёртую степень. То есть необходимо вычислить 2 -4

      Решение : как было сказано выше, 2 -4 = = = 0. 0625 .

      Ответ : 2 -4 = 0.0625 .

      В пятом веке до нашей эры древнегреческий философ Зенон Элейский сформулировал свои знаменитые апории, самой известной из которых является апория «Ахиллес и черепаха». Вот как она звучит:

      Допустим, Ахиллес бежит в десять раз быстрее, чем черепаха, и находится позади неё на расстоянии в тысячу шагов. За то время, за которое Ахиллес пробежит это расстояние, черепаха в ту же сторону проползёт сто шагов. Когда Ахиллес пробежит сто шагов, черепаха проползёт ещё десять шагов, и так далее. Процесс будет продолжаться до бесконечности, Ахиллес так никогда и не догонит черепаху.

      Это рассуждение стало логическим шоком для всех последующих поколений. Аристотель, Диоген, Кант, Гегель, Гильберт… Все они так или иначе рассматривали апории Зенона. Шок оказался настолько сильным, что «… дискуссии продолжаются и в настоящее время, прийти к общему мнению о сущности парадоксов научному сообществу пока не удалось. .. к исследованию вопроса привлекались математический анализ, теория множеств, новые физические и философские подходы; ни один из них не стал общепризнанным решением вопроса… » [Википедия, » Апории Зенона «]. Все понимают, что их дурят, но никто не понимает, в чем заключается обман.

      С точки зрения математики, Зенон в своей апории наглядно продемонстрировал переход от величины к . Этот переход подразумевает применение вместо постоянных. Насколько я понимаю, математический аппарат применения переменных единиц измерения либо ещё не разработан, либо его не применяли к апории Зенона. Применение же нашей обычной логики приводит нас в ловушку. Мы, по инерции мышления, применяем постоянные единицы измерения времени к обратной величине. С физической точки зрения это выглядит, как замедление времени до его полной остановки в момент, когда Ахиллес поравняется с черепахой. Если время останавливается, Ахиллес уже не может перегнать черепаху.

      Если перевернуть привычную нам логику, всё становится на свои места. Ахиллес бежит с постоянной скоростью. Каждый последующий отрезок его пути в десять раз короче предыдущего. Соответственно, и время, затрачиваемое на его преодоление, в десять раз меньше предыдущего. Если применять понятие «бесконечность» в этой ситуации, то правильно будет говорить «Ахиллес бесконечно быстро догонит черепаху».

      Как избежать этой логической ловушки? Оставаться в постоянных единицах измерения времени и не переходить к обратным величинам. На языке Зенона это выглядит так:

      За то время, за которое Ахиллес пробежит тысячу шагов, черепаха в ту же сторону проползёт сто шагов. За следующий интервал времени, равный первому, Ахиллес пробежит ещё тысячу шагов, а черепаха проползет сто шагов. Теперь Ахиллес на восемьсот шагов опережает черепаху.

      Этот подход адекватно описывает реальность без всяких логических парадоксов. Но это не полное решение проблемы. На Зеноновскую апорию «Ахиллес и черепаха» очень похоже утверждение Эйнштейна о непреодолимости скорости света. Эту проблему нам ещё предстоит изучить, переосмыслить и решить. И решение нужно искать не в бесконечно больших числах, а в единицах измерения.

      Другая интересная апория Зенона повествует о летящей стреле:

      Летящая стрела неподвижна, так как в каждый момент времени она покоится, а поскольку она покоится в каждый момент времени, то она покоится всегда.

      В этой апории логический парадокс преодолевается очень просто — достаточно уточнить, что в каждый момент времени летящая стрела покоится в разных точках пространства, что, собственно, и является движением. Здесь нужно отметить другой момент. По одной фотографии автомобиля на дороге невозможно определить ни факт его движения, ни расстояние до него. Для определения факта движения автомобиля нужны две фотографии, сделанные из одной точки в разные моменты времени, но по ним нельзя определить расстояние. Для определения расстояния до автомобиля нужны две фотографии, сделанные из разных точек пространства в один момент времени, но по ним нельзя определить факт движения (естественно, ещё нужны дополнительные данные для расчетов, тригонометрия вам в помощь). На что я хочу обратить особое внимание, так это на то, что две точки во времени и две точки в пространстве — это разные вещи, которые не стоит путать, ведь они предоставляют разные возможности для исследования.

      среда, 4 июля 2018 г.

      Очень хорошо различия между множеством и мультимножеством описаны в Википедии . Смотрим.

      Как видите, «во множестве не может быть двух идентичных элементов», но если идентичные элементы во множестве есть, такое множество называется «мультимножество». Подобную логику абсурда разумным существам не понять никогда. Это уровень говорящих попугаев и дрессированных обезьян, у которых разум отсутствует от слова «совсем». Математики выступают в роли обычных дрессировщиков, проповедуя нам свои абсурдные идеи.

      Когда-то инженеры, построившие мост, во время испытаний моста находились в лодке под мостом. Если мост обрушивался, бездарный инженер погибал под обломками своего творения. Если мост выдерживал нагрузку, талантливый инженер строил другие мосты.

      Как бы математики не прятались за фразой «чур, я в домике», точнее «математика изучает абстрактные понятия», есть одна пуповина, которая неразрывно связывает их с реальностью. Этой пуповиной являются деньги. Применим математическую теорию множеств к самим математикам.

      Мы очень хорошо учили математику и сейчас сидим в кассе, выдаем зарплату. Вот приходит к нам математик за своими деньгами. Отсчитываем ему всю сумму и раскладываем у себя на столе на разные стопки, в которые складываем купюры одного достоинства. Затем берем с каждой стопки по одной купюре и вручаем математику его «математическое множество зарплаты». Поясняем математику, что остальные купюры он получит только тогда, когда докажет, что множество без одинаковых элементов не равно множеству с одинаковыми элементами. Вот здесь начнется самое интересное.

      В первую очередь, сработает логика депутатов: «к другим это применять можно, ко мне — низьзя!». Дальше начнутся уверения нас в том, что на купюрах одинакового достоинства имеются разные номера купюр, а значит их нельзя считать одинаковыми элементами. Хорошо, отсчитываем зарплату монетами — на монетах нет номеров. Здесь математик начнет судорожно вспоминать физику: на разных монетах имеется разное количество грязи, кристаллическая структура и расположение атомов у каждой монеты уникально…

      А теперь у меня самый интересный вопрос: где проходит та грань, за которой элементы мультимножества превращаются в элементы множества и наоборот? Такой грани не существует — всё решают шаманы, наука здесь и близко не валялась.

      Вот смотрите. Мы отбираем футбольные стадионы с одинаковой площадью поля. Площадь полей одинакова — значит у нас получилось мультимножество. Но если рассматривать названия этих же стадионов — у нас получается множество, ведь названия разные. Как видите, один и тот же набор элементов одновременно является и множеством, и мультимножеством. Как правильно? А вот здесь математик-шаман-шуллер достает из рукава козырный туз и начинает нам рассказывать либо о множестве, либо о мультимножестве. В любом случае он убедит нас в своей правоте.

      Чтобы понять, как современные шаманы оперируют теорией множеств, привязывая её к реальности, достаточно ответить на один вопрос: чем элементы одного множества отличаются от элементов другого множества? Я вам покажу, без всяких «мыслимое как не единое целое» или «не мыслимое как единое целое».

      воскресенье, 18 марта 2018 г.

      Сумма цифр числа — это пляска шаманов с бубном, которая к математике никакого отношения не имеет. Да, на уроках математики нас учат находить сумму цифр числа и пользоваться нею, но на то они и шаманы, чтобы обучать потомков своим навыкам и премудростям, иначе шаманы просто вымрут.

      Вам нужны доказательства? Откройте Википедию и попробуйте найти страницу «Сумма цифр числа». Её не существует. Нет в математике формулы, по которой можно найти сумму цифр любого числа. Ведь цифры — это графические символы, при помощи которых мы записываем числа и на языке математики задача звучит так: «Найти сумму графических символов, изображающих любое число». Математики эту задачу решить не могут, а вот шаманы — элементарно.

      Давайте разберемся, что и как мы делаем для того, чтобы найти сумму цифр заданного числа. И так, пусть у нас есть число 12345. Что нужно сделать для того, чтобы найти сумму цифр этого числа? Рассмотрим все шаги по порядку.

      1. Записываем число на бумажке. Что же мы сделали? Мы преобразовали число в графический символ числа. Это не математическое действие.

      2. Разрезаем одну полученную картинку на несколько картинок, содержащих отдельные цифры. Разрезание картинки — это не математическое действие.

      3. Преобразовываем отдельные графические символы в числа. Это не математическое действие.

      4. Складываем полученные числа. Вот это уже математика.

      Сумма цифр числа 12345 равна 15. Вот такие вот «курсы кройки и шитья» от шаманов применяют математики. Но это ещё не всё.

      С точки зрения математики не имеет значения, в какой системе счисления мы записываем число. Так вот, в разных системах счисления сумма цифр одного и того же числа будет разной. В математике система счисления указывается в виде нижнего индекса справа от числа. С большим числом 12345 я не хочу голову морочить, рассмотрим число 26 из статьи про . Запишем это число в двоичной, восьмеричной, десятичной и шестнадцатеричной системах счисления. Мы не будем рассматривать каждый шаг под микроскопом, это мы уже сделали. Посмотрим на результат.

      Как видите, в разных системах счисления сумма цифр одного и того же числа получается разной. Подобный результат к математике никакого отношения не имеет. Это всё равно, что при определении площади прямоугольника в метрах и сантиметрах вы получали бы совершенно разные результаты.

      Ноль во всех системах счисления выглядит одинаково и суммы цифр не имеет. Это ещё один аргумент в пользу того, что . Вопрос к математикам: как в математике обозначается то, что не является числом? Что, для математиков ничего, кроме чисел, не существует? Для шаманов я могу такое допустить, но для ученых — нет. Реальность состоит не только из чисел.

      Полученный результат следует рассматривать как доказательство того, что системы счисления являются единицами измерения чисел. Ведь мы не можем сравнивать числа с разными единицами измерения. Если одни и те же действия с разными единицами измерения одной и той же величины приводят к разным результатам после их сравнения, значит это не имеет ничего общего с математикой.

      Что же такое настоящая математика? Это когда результат математического действия не зависит от величины числа, применяемой единицы измерения и от того, кто это действие выполняет.

      Открывает дверь и говорит:

      Ой! А это разве не женский туалет?
      — Девушка! Это лаборатория по изучению индефильной святости душ при вознесении на небеса! Нимб сверху и стрелочка вверх. Какой еще туалет?

      Женский… Нимб сверху и стрелочка вниз — это мужской.

      Если у вас перед глазами несколько раз в день мелькает вот такое вот произведение дизайнерского искусства,

      Тогда не удивительно, что в своем автомобиле вы вдруг обнаруживаете странный значок:

      Лично я делаю над собой усилие, чтобы в какающем человеке (одна картинка), увидеть минус четыре градуса (композиция из нескольких картинок: знак минус, цифра четыре, обозначение градусов). И я не считаю эту девушку дурой, не знающей физику. Просто у неё дугой стереотип восприятия графических образов. И математики нас этому постоянно учат. Вот пример.

      1А — это не «минус четыре градуса» или «один а». Это «какающий человек» или число «двадцать шесть» в шестнадцатеричной системе счисления. Те люди, которые постоянно работают в этой системе счисления, автоматически воспринимают цифру и букву как один графический символ.

      Основная цель

      Ознакомить учащихся со свойствами степеней с натуральными показателями и научить выполнять действия со степенями.

      Тема “ Степень и её свойства ” включает три вопроса:

      • Определение степени с натуральным показателем.
      • Умножение и деление степеней.
      • Возведение в степень произведения и степени.

      Контрольные вопросы

      1. Сформулируйте определение степени с натуральным показателем, большим 1. Приведите пример.
      2. Сформулируйте определение степени с показателем 1. Приведите пример.
      3. Каков порядок выполнения действий при вычислении значения выражения, содержащего степени?
      4. Сформулируйте основное свойство степени. Приведите пример.
      5. Сформулируйте правило умножения степеней с одинаковыми основаниями. Приведите пример.
      6. Сформулируйте правило деления степеней с одинаковыми основаниями. Приведите пример.
      7. Сформулируйте правило возведения в степень произведения. Приведите пример. Докажите тождество (ab) n = a n b n .
      8. Сформулируйте правило возведения степени в степень. Приведите пример. Докажите тождество (а m) n = а m n .

      Определение степени.

      Степенью числа a с натуральным показателем n , большим 1, называется произведение n множителей, каждый из которых равен а . Степенью числа а с показателем 1 называется само число а .

      Степень с основанием а и показателем n записывается так: а n . Читается “ а в степени n ”; “ n- я степень числа а ”.

      По определению степени:

      а 4 = а а а а

      . . . . . . . . . . . .

      Нахождение значения степени называют возведением в степень .

      1. Примеры возведения в степень:

      3 3 = 3 3 3 = 27

      0 4 = 0 0 0 0 = 0

      (-5) 3 = (-5) (-5) (-5) = -125

      25 ; 0,09 ;

      25 = 5 2 ; 0,09 = (0,3) 2 ; .

      27 ; 0,001 ; 8 .

      27 = 3 3 ; 0,001 = (0,1) 3 ; 8 = 2 3 .

      4. Найти значения выражений:

      а) 3 10 3 = 3 10 10 10 = 3 1000 = 3000

      б) -2 4 + (-3) 2 = 7
      2 4 = 16
      (-3) 2 = 9
      -16 + 9 = 7

      Вариант 1

      а) 0,3 0,3 0,3

      в) b b b b b b b

      г) (-х) (-х) (-х) (-х)

      д) (ab) (ab) (ab)

      2. Представьте в виде квадрата числа:

      3. Представьте в виде куба числа:

      4. Найти значения выражений:

      в) -1 4 + (-2) 3

      г) -4 3 + (-3) 2

      д) 100 — 5 2 4

      Умножение степеней.

      Для любого числа а и произвольных чисел m и n выполняется:

      a m a n = a m + n .

      Доказательство:

      Правило : При умножении степеней с одинаковыми основаниями основания оставляют прежним, а показатели степеней складывают.

      a m a n a k = a m + n a k = a (m + n) + k = a m + n + k

      а) х 5 х 4 = х 5 + 4 = х 9

      б) y y 6 = y 1 y 6 = y 1 + 6 = y 7

      в) b 2 b 5 b 4 = b 2 + 5 + 4 = b 11

      г) 3 4 9 = 3 4 3 2 = 3 6

      д) 0,01 0,1 3 = 0,1 2 0,1 3 = 0,1 5

      а) 2 3 2 = 2 4 = 16

      б) 3 2 3 5 = 3 7 = 2187

      Вариант 1

      1. Представить в виде степени:

      а) х 3 х 4 е) х 2 х 3 х 4

      б) а 6 а 2 ж) 3 3 9

      в) у 4 у з) 7 4 49

      г) а а 8 и) 16 2 7

      д) 2 3 2 4 к) 0,3 3 0,09

      2. Представить в виде степени и найти значение по таблице:

      а) 2 2 2 3 в) 8 2 5

      б) 3 4 3 2 г) 27 243

      Деление степеней.

      Для любого числа а0 и произвольных натуральных чисел m и n, таких, что m>n выполняется:

      a m: a n = a m — n

      Доказательство:

      a m — n a n = a (m — n) + n = a m — n + n = a m

      по определению частного:

      a m: a n = a m — n .

      Правило : При делении степеней с одинаковыми основаниями основание оставляют прежним, а из показателя степени делимого вычитают показатель степени делителя.

      Определение: Степень числа а, не равного нулю, с нулевым показателем равна единице :

      т.к. а n: a n = 1 при а0 .

      а) х 4:х 2 = х 4 — 2 = х 2

      б) у 8:у 3 = у 8 — 3 = у 5

      в) а 7:а = а 7:а 1 = а 7 — 1 = а 6

      г) с 5:с 0 = с 5:1 = с 5

      а) 5 7:5 5 = 5 2 = 25

      б) 10 20:10 17 = 10 3 = 1000

      в)

      г)

      д)

      Вариант 1

      1. Представьте в виде степени частное:

      2. Найдите значения выражений:

      Возведение в степень произведения.

      Для любых а и b и произвольного натурального числа n:

      (ab) n = a n b n

      Доказательство:

      По определению степени

      (ab) n =

      Сгруппировав отдельно множители а и множители b, получим:

      =

      Доказанное свойство степени произведения распространяется на степень произведения трех и более множителей.

      Например:

      (a b c) n = a n b n c n ;

      (a b c d) n = a n b n c n d n .

      Правило : При возведении в степень произведения возводят в эту степень каждый множитель и результат перемножают.

      1. Возвести в степень:

      а) (a b) 4 = a 4 b 4

      б) (2 х у) 3 =2 3 х 3 у 3 = 8 х 3 у 3

      в) (3 а) 4 = 3 4 а 4 = 81 а 4

      г) (-5 у) 3 = (-5) 3 у 3 = -125 у 3

      д) (-0,2 х у) 2 = (-0,2) 2 х 2 у 2 = 0,04 х 2 у 2

      е) (-3 a b c) 4 = (-3) 4 a 4 b 4 c 4 = 81 a 4 b 4 c 4

      2. Найти значение выражения:

      а) (2 10) 4 = 2 4 10 4 = 16 1000 = 16000

      б) (3 5 20) 2 = 3 2 100 2 = 9 10000= 90000

      в) 2 4 5 4 = (2 5) 4 = 10 4 = 10000

      г) 0,25 11 4 11 = (0,25 4) 11 = 1 11 = 1

      д)

      Вариант 1

      1. Возвести в степень:

      б) (2 а с) 4

      д) (-0,1 х у) 3

      2. Найти значение выражения:

      б) (5 7 20) 2

      Возведение в степень степени.

      Для любого числа а и произвольных натуральных чисел m и n:

      (а m) n = а m n

      Доказательство:

      По определению степени

      (а m) n =

      Правило: При возведении степени в степень основание оставляют тем же, а показатели перемножают .

      1. Возвести в степень:

      (а 3) 2 = а 6 (х 5) 4 = х 20

      (у 5) 2 = у 10 (b 3) 3 = b 9

      2. Упростите выражения:

      а) а 3 (а 2) 5 = а 3 а 10 = а 13

      б) (b 3) 2 b 7 = b 6 b 7 = b 13

      в) (х 3) 2 (х 2) 4 = х 6 х 8 = х 14

      г) (у у 7) 3 = (у 8) 3 = у 24

      а)

      б)

      Вариант 1

      1. Возвести в степень:

      а) (а 4) 2 б) (х 4) 5

      в) (у 3) 2 г) (b 4) 4

      2. Упростите выражения:

      а) а 4 (а 3) 2

      б) (b 4) 3 b 5+

      в) (х 2) 4 (х 4) 3

      г) (у у 9) 2

      3. Найдите значение выражений:

      Приложение

      Определение степени.

      Вариант 2

      1ю Запишите произведение в виде степени:

      а) 0,4 0,4 0,4

      в) а а а а а а а а

      г) (-у) (-у) (-у) (-у)

      д) (bс) (bс) (bс)

      2. Представьте в виде квадрата числа:

      3. Представьте в виде куба числа:

      4. Найти значения выражений:

      в) -1 3 + (-2) 4

      г) -6 2 + (-3) 2

      д) 4 5 2 – 100

      Вариант 3

      1. Запишите произведение в виде степени:

      а) 0,5 0,5 0,5

      в) с с с с с с с с с

      г) (-х) (-х) (-х) (-х)

      д) (ab) (ab) (ab)

      2. Представьте в виде квадрата числа: 100 ; 0,49 ; .

      3. Представьте в виде куба числа:

      4. Найти значения выражений:

      в) -1 5 + (-3) 2

      г) -5 3 + (-4) 2

      д) 5 4 2 — 100

      Вариант 4

      1. Запишите произведение в виде степени:

      а) 0,7 0,7 0,7

      в) х х х х х х

      г) (-а) (-а) (-а)

      д) (bс) (bс) (bс) (bc)

      2. Представьте в виде квадрата числа:

      3. Представьте в виде куба числа:

      4. Найти значения выражений:

      в) -1 4 + (-3) 3

      г) -3 4 + (-5) 2

      д) 100 — 3 2 5

      Умножение степеней.

      Вариант 2

      1. Представить в виде степени:

      а) х 4 x 5 е) х 3 х 4 х 5

      б) а 7 а 3 ж) 2 3 4

      в) у 5 у з) 4 3 16

      г) а а 7 и) 4 2 5

      д) 2 2 2 5 к) 0,2 3 0,04

      2. Представить в виде степени и найти значение по таблице:

      а) 3 2 3 3 в) 16 2 3

      б) 2 4 2 5 г) 9 81

      Вариант 3

      1. Представить в виде степени:

      а) а 3 а 5 е) у 2 у 4 у 6

      б) х 4 х 7 ж) 3 5 9

      в) b 6 b з) 5 3 25

      г) у у 8 и) 49 7 4

      д) 2 3 2 6 к) 0,3 4 0,27

      2. Представить в виде степени и найти значение по таблице:

      а) 3 3 3 4 в) 27 3 4

      б) 2 4 2 6 г) 16 64

      Вариант 4

      1. Представить в виде степени:

      а) а 6 а 2 е) х 4 х х 6

      б) х 7 х 8 ж) 3 4 27

      в) у 6 у з) 4 3 16

      г) х х 10 и) 36 6 3

      д) 2 4 2 5 к) 0,2 2 0,008

      2. Представить в виде степени и найти значение по таблице:

      а) 2 6 2 3 в) 64 2 4

      б) 3 5 3 2 г) 81 27

      Деление степеней.

      Вариант 2

      1. Представьте в виде степени частное:

      2. Найдите значения выражений.

      можно найти с помощью умножения. Например: 5+5+5+5+5+5=5х6. О таком выражении говорят, что сумму равных слагаемых свернули в произведение. И наоборот, если читать это равенство справа налево, получаем, что мы развернули сумму равных слагаемых. Аналогично можно сворачивать произведение нескольких равных множителей 5х5х5х5х5х5=5 6 .

      То есть вместо умножения шести одинаковых множителей 5х5х5х5х5х5 пишут 5 6 и говорят «пять в шестой степени».

      Выражение 5 6 — это степенью числа, где:

      5 — основание степени;

      6 — показатель степени.

      Действия, с помощью которых произведение равных множителей сворачивают в степень, называют возведением в степень.

      В общем виде степень с основанием «a» и показателем «n» записывается так

      Возвести число a в степень n — значит найти произведение n множителей, каждый из которых равен а

      Если основание степени «а» равно 1, то значение степени при любом натуральном n будет равно 1. Например, 1 5 =1, 1 256 =1

      Если возвести число «а» возвести в первую степень , то получим само число a: a 1 = a

      Если возвести любое число в нулевой степень , то в результате вычислений получим один. a 0 = 1

      Особыми считают вторую и третью степень числа. Для них придумали названия: вторую степень называют квадратом числа , третью — кубом этого числа.

      В степень можно возводить любое число — положительное, отрицательное или нуль. При этом не пользуются следующими правилами:

      При нахождении степени положительного числа получается положительное число .

      При вычислениях нуля в натуральной степени получаем ноль.

      х m · х n = х m + n

      например: 7 1.7 · 7 — 0.9 = 7 1.7+(- 0.9) = 7 1.7 — 0.9 = 7 0.8

      Чтобы разделить степени с одинаковыми основаниями основание не меняем, а показатели степеней вычитаем :

      х m / х n = х m — n , где, m > n,

      например: 13 3.8 / 13 -0.2 = 13 (3.8 -0.2) = 13 3.6

      При расчетах возведения степени в степень основание не меняем, а показатели степеней умножаем друг на друга.

      (у m ) n = у m · n

      например: (2 3) 2 = 2 3·2 = 2 6

      (х · у) n = х n · у m ,

      например:(2·3) 3 = 2 n · 3 m ,

      При выполнении расчетов по возведению в степень дроби мы в данную степень возводим числитель и знаменатель дроби

      (х / у) n = х n / у n

      например: (2 / 5) 3 = (2 / 5) · (2 / 5) · (2 / 5) = 2 3 / 5 3 .

      Последовательность выполнения расчетов при работе с выражениями содержащими степень.

      При выполнении расчетов выражений без скобок, но содержащих степени, в первую очередь производят возведение в степень, потом действия умножение и деление, и лишь потом операции сложения и вычитания.

      Если необходимо вычислить выражение содержащие скобки, то сначала в указанном выше порядке делаем вычисления в скобках, а потом оставшиеся действия в том же порядке слева направо.

      Очень широко в практических вычислениях для упрощения расчетов используют готовые таблицы степеней.

      Табличка на двери

      Как возводить в степень в Экселе

      Содержание

      • Возведение чисел
        • Способ 1: возведение с помощью символа
        • Способ 2: применение функции
        • Способ 3: возведение в степень через корень
        • Способ 4: запись числа со степенью в ячейке
      • Вопросы и ответы

      Возведение числа в степень является стандартным математическим действием. Оно применяется в различных расчетах, как в учебных целях, так и на практике. 6

    8. Жмем на кнопку Enter. Как видим, расчет был выполнен корректно. Так как в ячейке A2 находилось число 7, то результат вычисления составил 117649.
    9. Если мы хотим возвести в одну и ту же степень целый столбец чисел, то не обязательно записывать формулу для каждого значения. Достаточно записать её для первой строки таблицы. Затем просто нужно навести курсор на нижний правый угол ячейки с формулой. Появится маркер заполнения. Зажимаем левую кнопку мыши и протягиваем его к самому низу таблицы.

    Как видим, все значения нужного интервала были возведены в указанную степень.

    Данный способ максимально прост и удобен, и поэтому так популярен у пользователей. Именно он применяется в подавляющем большинстве случаев вычислений.

    Урок: Работа с формулами в Excel

    Урок: Как сделать автозаполнение в Excel

    Способ 2: применение функции

    В Экселе имеется также специальная функция для проведения данного расчета. Она так и называется – СТЕПЕНЬ. Её синтаксис выглядит следующим образом:

    =СТЕПЕНЬ(число;степень)

    Рассмотрим её применение на конкретном примере.

    1. Кликаем по ячейке, куда планируем выводить результат расчета. Жмем на кнопку «Вставить функцию».
    2. Открывается Мастер функций. В списке элементов ищем запись «СТЕПЕНЬ». После того как находим, выделяем её и жмем на кнопку «OK».
    3. Открывается окно аргументов. У данного оператора два аргумента – число и степень. Причем в качестве первого аргумента может выступать, как числовое значение, так и ячейка. То есть, действия производятся по аналогии с первым способом. Если в качестве первого аргумента выступает адрес ячейки, то достаточно поставить курсор мыши в поле «Число», а потом кликнуть по нужной области листа. После этого, числовое значение, хранящееся в ней, отобразится в поле. Теоретически в поле «Степень» в качестве аргумента тоже можно использовать адрес ячейки, но на практике это редко применимо. После того, как все данные введены, для того, чтобы произвести вычисление, жмем на кнопку «OK».

    Вслед за этим результат вычисления данной функции выводится в место, которое было выделено ещё в первом шаге описываемых действий.

    Кроме того, окно аргументов можно вызвать, перейдя во вкладку «Формулы». На ленте следует нажать кнопку «Математические», расположенную в блоке инструментов «Библиотека функций». В открывшемся списке доступных элементов нужно выбрать «СТЕПЕНЬ». После этого запустится окно аргументов этой функции.

    Пользователи, которые имеют определенный опыт, могут не вызывать Мастер функций, а просто вводить формулу в ячейку после знака «=», согласно её синтаксису.

    Данный способ более сложный, чем предыдущий. Его применение может быть обосновано, если расчет нужно произвести в границах составной функции, состоящей из нескольких операторов.

    Урок: Мастер функций в Excel

    Способ 3: возведение в степень через корень

    Конечно, данный способ не совсем обычный, но к нему тоже можно прибегнуть, если нужно возвести число в степень 0,5. Разберем этот случай на конкретном примере.

    Нам нужно возвести 9 в степень 0,5 или по-другому — ½.

    1. Выделяем ячейку, в которую будет выводиться результат. Кликаем по кнопке «Вставить функцию».
    2. В открывшемся окне Мастера функций ищем элемент КОРЕНЬ. Выделяем его и жмем на кнопку «OK».
    3. Открывается окно аргументов. Единственным аргументом функции КОРЕНЬ является число. Сама функция выполняет извлечение квадратного корня из введенного числа. Но, так как квадратный корень тождественен возведению в степень ½, то нам данный вариант как раз подходит. В поле «Число» вводим цифру 9 и жмем на кнопку «OK».
    4. После этого, в ячейке рассчитывается результат. В данном случае он равен 3. Именно это число и является результатом возведения 9 в степень 0,5.

    Но, конечно, к данному способу расчета прибегают довольно редко, используя более известные и интуитивно понятные варианты вычислений.

    Урок: Как посчитать корень в Экселе

    Способ 4: запись числа со степенью в ячейке

    Этот способ не предусматривает проведения вычислений по возведению. Он применим только тогда, когда нужно просто записать число со степенью в ячейке.

    1. Форматируем ячейку, в которую будет производиться запись, в текстовый формат. Выделяем её. Находясь во вкладке em«Главная» на ленте в блоке инструментов «Число», кликаем по выпадающему списку выбора формата. Жмем по пункту «Текстовый».
    2. В одной ячейке записываем число и его степень. Например, если нам нужно написать три во второй степени, то пишем «32».
    3. Ставим курсор в ячейку и выделяем только вторую цифру.
    4. Нажатием сочетания клавиш Ctrl+1 вызываем окно форматирования. Устанавливаем галочку около параметра «Надстрочный». Жмем на кнопку «OK».
    5. После этих манипуляций на экране отразится заданное число со степенью.

    Внимание! Несмотря на то, что визуально в ячейке будет отображаться число в степени, Excel воспринимает его как обычный текст, а не числовое выражение. Поэтому для расчетов такой вариант применять нельзя. ». В отдельных случаях можно применить функцию СТЕПЕНЬ. Если вам нужно возвести число в степень 0,5, то существует возможность воспользоваться функцией КОРЕНЬ. Если же пользователь хочет визуально отобразить степенное выражение без вычислительных действий, то тут на помощь придет форматирование.

    Возведение степень в экселе функция. Возведение числа в степень в Microsoft Excel

    Продолжаем обзор математических функций и возможность. Сегодня на очереди формула из простейших — степень в Excel. Возведение в степень (корень) функцией или простым обозначениями, отрицательная степень. Как красиво записать степень, тоже будет здесь. Все в принципе просто, но это не значит, что об этом не нужно написать статейку. Тем более одной большой статьи, охватывающей все вопросы по степеням я не нашел. Так что читаем эту статью здесь:

    Все мы знаем что степень обозначает перемножение одинаковых чисел, столько раз, сколько степень у числа. Т.е. 2 в 3 степени = 8 = 2 * 2 * 2. -3

    Как вам больше нравится?

    Как найти корень числа в Excel? Функция Корень

    Как мы знаем, что квадратный корень числа — это число в степени 0,5 (1/2). Поэтому найти корень числа также просто как и возвести в любую степень. Если хотите извлечь из четырех корень из 2, то запишите =СТЕПЕНЬ(4;0,5). Или для этого есть специальная функция =КОРЕНЬ(Число). Показываю сразу несколько способов извлечь корень числа.

    Как красиво записать степень в Excel в подписи к расчетам?

    Сейчас поговорим, как красиво записать подписи к формулам. Под красивой записью мы подразумеваем т.н. надстрочный шрифт. Как здесь:

    Так сделать очень просто, выделите в строке формул или в ячейке число, обозначающее степень (то что надо сделать маленьким).

    При работе с математическими выражениями периодически появляется потребность в возведении числа в какую-нибудь степень. Условно сделать это можно двумя способами: непосредственно написать число, уже возведенное в степень, или же поставить рядом с ним сам знак степени. ». Для этого зажмите клавишу Shift и нажмите кнопку 6 на верхнем цифровом ряду.

  2. Введите степень, в которую хотите возвести число, то есть 4.
  3. Нажмите Enter.

Сразу после этого в ячейке появится результат данной математической операции. Теперь вы в курсе, как поставить степень в «Экселе», чтобы число сразу же возводилось в нее.

Способ 2: с помощью функции

Для достижения поставленного результата можно прибегнуть и к помощи функции, кстати, называется она соответствующе — СТЕПЕНЬ. Способ, как поставить степень в «Экселе» с помощью функции, не отличается особой сложностью:

  1. Выделите ячейку, в которую хотите записать выражение.
  2. Нажмите по кнопке «Вставить функцию».
  3. В появившемся окне «Мастера функций» найдите в списке строку «СТЕПЕНЬ», выделите ее и нажмите ОК.
  4. Перед вами появится новое окно с двумя полями для ввода. В первое вам необходимо ввести число, которое собираетесь возвести в степень, а во второе непосредственно саму степень. Кстати, вместо числа вы можете указать ячейку с числовым выражением, тогда именно оно будет возведено в степень.
  5. Нажмите ОК.

Теперь вы знаете второй способ, как поставить степень в «Экселе», но что, если вам необходимо не возвести число, а просто указать его степень в текстовом формате? В этом случае переходите к третьему способу.

Способ 3: написание степени в ячейке таблицы

Если вы хотите непосредственно в ячейке указать степень числа, тогда необходимо использовать надстрочный знак. На самом деле выполнение такой записи не является чрезвычайно сложным, просто алгоритм действия для этого не совсем понятен рядовому пользователю, ведь перед самой записью потребуется сделать еще некоторое приготовление. Но обо всем по порядку. Давайте рассмотрим все на примере, чтобы сразу было понятно, как это работает. Разбирать будем все тот же случай, когда нам требуется возвести число 3 в четвертую степень.

  1. Выделите ячейку, в которой хотите сделать запись.
  2. Измените ее формат на текстовый. Для этого, находясь на главной вкладке, на панели инструментов нажмите по выпадающему списку, расположенному в группе «Число». В появившемся списке выберите пункт «Текстовый».
  3. Теперь все готово для того, чтобы запись в ячейке отобразилась корректно. Итак, введите в нее число 34. Почему именно его? Да потому, что 4 — это степень числа 3, которую мы сейчас будем видоизменять.
  4. Выделите в ячейке непосредственно саму цифру 4.
  5. Нажмите на клавиатуре горячие клавиши Ctrl+1.
  6. Появится окно «Формат ячеек». В нем вам необходимо в области «Видоизменение» установить отметку напротив пункта «Надстрочный».
  7. Нажмите ОК.

Сразу после этого выражение будет записано так, как вам нужно. Как видим, осуществить запись правильного формата не так уж и сложно, как это может показаться на первый взгляд.

Заключение

Теперь вы знаете три способа, как работать со степенью в программе Excel. Два из них подразумевают непосредственное возведение в заданную степень, что очень удобно при вычислениях. Однако если вам необходимо записать число непосредственно со степенью в текстовом формате, тогда воспользуйтесь третьим методом, который довольно прост в выполнении.

Часто пользователям необходимо возвести число в степень. Как правильно сделать это с помощью «Экселя»?

В этой статье мы попробуем разобраться с популярными вопросами пользователей и дать инструкцию по правильному использованию системы. MS Office Excel позволяет выполнять ряд математических функций: от самых простых до сложнейших. Это универсальное программное обеспечение рассчитано на все случаи жизни.

Как возвести в степень в Excel?

Перед поиском необходимой функции обратите внимание на математические законы:

  1. Число «1» в любой степени будет оставаться «1».
  2. Число «0» в любой степени будет оставаться «0».
  3. Любое число, возведенное в нулевую степень, равняется единице.
  4. Любое значение «А» в степени «1» будет равняться «А».

Примеры в Excel:

Вариант №1.

».

Мы возвели 8 в «квадрат» (т.е. ко второй степени) и получили в ячейке «А2» результат вычисления.



Вариант №2. С использованием функции

В Microsoft Office Excel есть удобная функция «СТЕПЕНЬ», которую вы можете активизировать для осуществления простых и сложных математических расчетов.

Функция выглядит следующим образом:

СТЕПЕНЬ(число;степень)

ВНИМАНИЕ!

  1. Цифры для этой формулы указываются без пробелов и других знаков.
  2. Первая цифра – значение «число». Это основание (т.е. цифра, которую мы возводим). Microsoft Office Excel допускает введение любого вещественного числа.
  3. Вторая цифра – значение «степень». Это показатель, в который мы возводим первую цифру.
  4. Значения обоих параметров могут быть меньше нуля (т.е. со знаком «-»).

Формула возведения в степень в Excel

Примеры использования функции СТЕПЕНЬ().

С использованием мастера функций:

Если лишние клики вы считаете сомнительным удовольствием, предлагаем еще один простой вариант. (1/n)- где a-число; n-степень:

Или через такую функцию: =СТЕПЕНЬ(32;1/5)

В аргументах формулы и функции можно указывать ссылки на ячейки вместо числа.

Как в Excel написать число в степени?

Часто вам важно, чтобы число в степени корректно отображалось при распечатывании и красиво выглядело в таблице. Как в Excel написать число в степени? Здесь необходимо использовать вкладку «Формат ячеек». В нашем примере мы записали цифру «3» в ячейку «А1», которую нужно представить в -2 степени.

Последовательность действий следующая:


Пользоваться возможностями Excel просто и удобно. С ними вы экономите время на осуществлении математических подсчетов и поисках необходимых формул.

Возведение числа в степень является стандартным математическим действием. Оно применяется в различных расчетах, как в учебных целях, так и на практике. У программы Excel имеются встроенные инструменты для подсчета данного значения. Давайте посмотрим, как ими пользоваться в различных случаях. » можно возводить не только обычные числа, но и данные, содержащиеся в определенном диапазоне листа.

Возведем в шестую степень содержимое ячейки A2.


Как видим, все значения нужного интервала были возведены в указанную степень.

Данный способ максимально прост и удобен, и поэтому так популярен у пользователей. Именно он применяется в подавляющем большинстве случаев вычислений.

Способ 2: применение функции

В Экселе имеется также специальная функция для проведения данного расчета. Она так и называется – СТЕПЕНЬ . Её синтаксис выглядит следующим образом:

СТЕПЕНЬ(число;степень)

Рассмотрим её применение на конкретном примере.


Вслед за этим результат вычисления данной функции выводится в место, которое было выделено ещё в первом шаге описываемых действий.

Кроме того, окно аргументов можно вызвать, перейдя во вкладку «Формулы» . На ленте следует нажать кнопку «Математические» , расположенную в блоке инструментов «Библиотека функций» . В открывшемся списке доступных элементов нужно выбрать «СТЕПЕНЬ» . После этого запустится окно аргументов этой функции.

Пользователи, которые имеют определенный опыт, могут не вызывать Мастер функций , а просто вводить формулу в ячейку после знака «=» , согласно её синтаксису.

Данный способ более сложный, чем предыдущий. Его применение может быть обосновано, если расчет нужно произвести в границах составной функции, состоящей из нескольких операторов.

Способ 3: возведение в степень через корень

Конечно, данный способ не совсем обычный, но к нему тоже можно прибегнуть, если нужно возвести число в степень 0,5. Разберем этот случай на конкретном примере.

Нам нужно возвести 9 в степень 0,5 или по-другому — ½.


Но, конечно, к данному способу расчета прибегают довольно редко, используя более известные и интуитивно понятные варианты вычислений.

Способ 4: запись числа со степенью в ячейке

Этот способ не предусматривает проведения вычислений по возведению. » . В отдельных случаях можно применить функцию СТЕПЕНЬ . Если вам нужно возвести число в степень 0,5, то существует возможность воспользоваться функцией КОРЕНЬ . Если же пользователь хочет визуально отобразить степенное выражение без вычислительных действий, то тут на помощь придет форматирование.

Одним из наиболее частых математических действий, применяемых в инженерных и других вычислениях, является возведение числа во вторую степень, которую по-другому называют квадратной. Например, данным способом рассчитывается площадь объекта или фигуры. К сожалению, в программе Excel нет отдельного инструмента, который возводил бы заданное число именно в квадрат. Тем не менее, эту операцию можно выполнить, использовав те же инструменты, которые применяются для возведения в любую другую степень. Давайте выясним, как их следует использовать для вычисления квадрата от заданного числа.

Как известно, квадрат числа вычисляется его умножением на самого себя. Данные принципы, естественно, лежат в основе вычисления указанного показателя и в Excel. » . При этом, в качестве объекта, который будет возведен в квадрат, можно использовать число или ссылку на ячейку, где данное числовое значение расположено.

Общий вид формулы для возведения в квадрат следующий:

В ней вместо «n» нужно подставить конкретное число, которое следует возвести в квадрат.

Посмотрим, как это работает на конкретных примерах. Для начала возведем в квадрат число, которое будет составной частью формулы.


Теперь давайте посмотрим, как возвести в квадрат значение, которое расположено в другой ячейке.


Способ 2: использование функции СТЕПЕНЬ

Также для возведения числа в квадрат можно использовать встроенную функцию Excel СТЕПЕНЬ . Данный оператор входит в категорию математических функций и его задачей является возведение определенного числового значения в указанную степень. Синтаксис у функции следующий:

СТЕПЕНЬ(число;степень)

Аргумент «Число» может представлять собой конкретное число или ссылку на элемент листа, где оно расположено.

Аргумент «Степень» указывает на степень, в которую нужно возвести число. Так как перед нами поставлен вопрос возведения в квадрат, то в нашем случае данный аргумент будет равен 2 .

Теперь посмотрим на конкретном примере, как производится возведение в квадрат с помощью оператора СТЕПЕНЬ .


Также для решения поставленной задачи вместо числа в виде аргумента можно использовать ссылку на ячейку, в которой оно расположено.


Формула экспонентов

 – Что такое формула экспонентов? Примеры

Показатель степени числа означает, сколько раз любое число умножается само на себя. Существуют различные формулы показателей степени, используемые для решения уравнений. Показатели важны, потому что они помогают в представлении продуктов, где число повторяется само по себе много раз. Давайте узнаем о формулах экспонент с несколькими решенными примерами в конце.

Что такое формулы экспоненты?

Формулы экспоненты относятся к формулам, которые помогают вычислять экспоненты. Показатель степени числа представлен в виде: x n , что означает, что x умножается сам на себя n раз.   Здесь

  • x называется «основой»
  • .
  • n называется «показатель степени» или «степень»
  • x n читается как «x в степени n» (или) «x в степени n»

Формулы степени

Формулы степени выражены как:

  • a = 1
  • а 1  = а
  • a м  × a = a m+n
  • a / a = a m−n
  • а − м = 1/а м
  • (a м ) = a mn
  • (ab) м  = a м b м
  • (a/b) м  = a м /b м

Давайте лучше разберемся в формулах экспонент на нескольких решенных примерах.

 

Разбивайте сложные концепции с помощью простых визуальных средств.

Математика больше не будет сложным предметом, особенно когда вы понимаете концепции с помощью визуализаций.

Записаться на бесплатный пробный урок

Примеры с использованием экспонентных формул

Пример 1: В лесу на каждом дереве около 5 7 листьев, а в лесу около 5 3 деревьев. Используя формулу показателей, найдите общее количество листьев.

Решение: 

Найти: Общее количество листьев.

Количество деревьев в лесу = 5 3

Количество листьев на каждом дереве = 5 7 (дано)

Используя формулу показателей, x+y

Установление значений,

5 3 x 5 7 = 5 3+7

5 3 x 5 7 = 5 10

Ответ: общее количество листьев 5 10 .

Пример 2: Размеры шкафа: x 5 дюймов, y 3 дюймов и x 8 дюймов. Найдите его объем.

Решение: 

Найти: объем гардероба.

Размеры шкафа: длина (д) = x 5 дюймов, ширина (ш) = y 3 дюймов, высота (h) = x 8 дюймов (данные)

Использование формулы показателей ,

A x x A y = A x+y

Установление значений,

том = x 5 × x 8 × y 3 = x 13 × y 3 3 × y 3 = x 13 × y

Объем = x 13  × y 3

Ответ: Объем гардероба равен x 13  × y 3 .

Пример 3: Определите значение x -5/2  при x = 3.

Решение:

Чтобы найти: значение x -5/2  

Дано: x = 3

Используя формулу экспонент,

x -5/2 = (3) -803

= (1/3) 5/2

= (1/3 × 1/3 × 1/3 × 1/3 × 1/3) 1/2

= (1/243) 1/2

= √(1/243) = 1/9√3.

Ответ: Значение x -5/2  при x = 3 равно 1/9√3.

Часто задаваемые вопросы о формулах экспоненты

Что такое формулы экспоненты в математике?

Мы знаем, что показатель степени числа представлен в виде: x n  (n — показатель степени). Другими словами, вы можете сказать, что показатели степени — это надстрочные числа. Формулы экспонент – это формулы, которые помогают решать задачи, связанные с экспонентами. Некоторые важные формулы показателей представлены в виде

  • a 0 = 1
  • .
  • а 1  = а
  • a м  × a = a m+n
  • а м / а = а m−n
  • а − м = 1/а м
  • (a м ) = a mn
  • (ab) м  = a м b м
  • (a/b) м  = a м /b м

Каковы применения формул экспоненты?

Формулы экспонентов имеют широкий спектр применения:

  • научные шкалы, такие как шкала pH или шкала Рихтера.
  • расчет площади, объема и другие подобные задачи.
  • сокращает то, что в противном случае было бы очень утомительно писать.
  • используется в компьютерных играх, мерных весах.
  • Наука, инженерия, экономика, бухгалтерский учет, финансы.
  • Экспоненты часто используются для описания памяти компьютера.
  • Изготовление лекарств в лаборатории.

Как использовать формулы экспоненты?

Формулы степени просты в использовании

  • Шаг 1: Проверьте заданные параметры.
  • Шаг 2: Найдите подходящую формулу экспоненты.
  • Шаг 3: Подставьте заданные значения (основание и степень) в формулу.

Какие компоненты формул экспоненты?

Формулы экспонент включают основания, степени и математические символы. Показатель степени числа представлен как x n , где «x» — основание, а «n» — степень.

Экспоненты и формулы степеней — Законы экспонентов Формулы и решенные примеры

Если вы снова и снова задаетесь вопросом, что такое экспонента в математике, то знайте, что экспонента, также известная как степень, представляет собой математический способ выражения числа, умноженного на себя определенное определенное число раз. С математической точки зрения, когда мы пишем нецелое число а, на самом деле это 1 , обозначаемое как — в степени 1.

a 2 = a*a

a 3 = а*а*а

а 4 = а*а*а*а

а 5 = а*а*а*а

:

:

а 8 9900 а*а*а*а*а*а. . . n раз.

Ниже приведена стандартная формула экспоненты и степени для решения задач на экспоненты.

(am)n = (an)m = a(mn)

Данная диаграмма разбивает экспоненциальное выражение на части, точно определяя, какое число является экспоненциальной степенью фактора.

Что такое сила в математике?

В математике степень определяется как выражение, представляющее многократное умножение одного и того же заданного числа. Записывается как «возведение числа в степень любого другого числа».

Например, 7 × 7 × 7 × 7 = 2041, что также можно записать как 7 4 = 2041, что означает, что число «7» нужно умножить четыре раза само на себя, чтобы получить число «2041». . Другими словами, можно сказать, что число «4», возведенное в степень 4, или число «7», возведенное в степень 4 9.0007-я -я сила», что дает нам число «7». Здесь число «4» называется основанием, а «4» — степенью или показателем степени.

Что такое показатель степени в математике?

Показатель степени в математике определяется как положительное или отрицательное число, которое описывает степень, в которую возводится основание числа. Другими словами, он указывает, сколько раз нужно использовать число в процессе умножения.

 Например, 6 3 = 6 × 6 × 6 равно 216. Где базовое число равно «6», которое используется 3 раза при умножении. Следовательно, мы умножаем число «6» три раза само на себя, чтобы получить число «216». В геометрии куб и квадрат являются двумя наиболее часто используемыми показателями степени.

Tabular Representation Clarifying Definitions

Expressions

Long Hand Expression

Base

Exponent/Power

Value

2 5

2×2×2×2×2

2

5

3 92904 3 9231

5 3

5×5×5

5

3

125

3 5

3×3 ×3×3×3

3

5

243

7 4

7×7×7×7

7

4

2401

В задачах на чтение Математические выражения с экспоненциальными степенями, такие как74, часто произносятся как «семь в четвертой степени». Опять же, экспоненциальные выражения, такие как 74, часто читаются как «4-я степень 7».

(изображение будет загружено в ближайшее время)

Законы экспонент Формулы

Существуют различные законы экспонент, которые вы должны практиковать и помнить, чтобы полностью понять экспоненциальные концепции. Следующий экспоненциальный закон подробно описан с примерами экспоненциальных степеней, радикалов и корней. 9n}\]

\[a\sqrt{c}+b\sqrt{c}\] = \[(a+b)\sqrt{c}\]

\[\sqrt{a}+\sqrt {b}\] ≠ \[\sqrt{(a+b)}\]

\[\frac{ax}{y}=\sqrt[y]{ax}\]

Свойства экспонент и радикалов

Exponent and Radical Rules

Example

Notes

xᵐ=x⋅x⋅x⋅x…..(m times)

2³=2⋅2⋅2=8

92=\frac{9}{4}\]

Если основание возводится в отрицательную степень, это означает, что берется обратная величина, а показатель степени становится положительным.

\[a\sqrt{x}\times b\sqrt{y}=ab\sqrt{xy}\]

 


\[2\sqrt{4\3}\times sqrt{5}=8\sqrt{15}\]

Перемножая два подкоренных члена, вы можете умножить то, что находится снаружи (коэффициент), и то, что внутри (коренное число). Это можно сделать только в том случае, если корни (индексы) такие же, как квадратный корень или кубический корень. 92=16; x=\pm 4\]

Требуется включить в уравнение с четным показателем степени как положительное, так и отрицательное решение при извлечении четного корня. Поскольку и положительный корень, и отрицательный корень работают, когда возводятся в эту степень. Знак квадратного корня дает только положительные решения.


 

Решение экспоненциальных уравнений с использованием законов показательного уравнения, следующее правило поможет вам четко понять различные способы нахождения неизвестного значения в показательном уравнении.

92-x-2=0\]         ……….3

Применить правило разложения, упростить и решить

Получаем

\[(x-2)(x+1)\]

\[x= +2,-1\].

Решенные примеры формул показателей и степеней

Пример 1:

If m 2 +n 2 +o 2 = mn+no+n, упростить [y 0 0 0 8

/y] m-n

× [y n /y o ] n-o × [y o /y m ] o-m

Solution1:

с использованием M A /N B = M A-B , мы получаем

→ (Y M-N ) M-N × (Y N-O ) N-O × (Y N-O ) N-o × (Y O-M (Y ) N-o × (Y N-O ) ) o-m

По формуле o²−2no)×y(o²+m²−2om)

Математика

m a . m b =m a+b

→ y(m²+n²−2mn+n²+o²−2no+o²+m²−2om)

→ y2(m²+n²+o²−(mn+no+om))

→ y [2(0 )]

→ Y 0 = 1

Пример 2: Найдите Y, если

3 2y-1 +3 2y +1 = 270

Solution2:

Мы сначала выпустим сначала. общий термин, с которым мы получаем

→ 3 2y-1 (1+3 2 )

Видим, что здесь мы используем формулу для любого нецелого числа a m+n = a м . a n при выражении 3 2y+1 как произведения 3 2y-1 и 3 2 .

→ 3 2Y-1 (10) = 270

→ 3 2Y-1 = 27

→ 3 2y-1 = 3 3

→ 2y -1 = 3

→ y = 2.

Формула экспоненты — GeeksforGeeks

Экспонента является одной из важных основ математики. Экспоненты используются в различных формулах, таких как ряды, биномиальное разложение и многие другие. Различные формулы экспоненты используются в различных областях математики. Формулы экспоненты очень просты и полезны. Давайте узнаем о показателях и их формулах. Экспоненты — это степени любой переменной или константы.

Показатель степени 

Если любое число или переменную (x) умножить n раз, результатом будет x n . Тогда n называется показателем степени x. x.x.x.x.x.x … n times = x n   , тогда x — основание, а n — показатель степени этого основания. Экспонента — это степень числа. Оно умножается само на себя. Экспонента определяет, сколько раз число умножается само на себя. Пример, 2.2.2 = 2 3 , основание = 2, показатель степени = 3.

Экспоненты Формулы  

(M — N) 9048 904877777777777777777777777777777777777777777777777777777777777777777777777777777777777777777777777777777 (M — N) n 888 . m ] n = x mn

  Exponents Formulas

n times product exponent formula x. x.x.x … n times = x n   
Multiplication Rule x m . x N = x (M + N)
Правило дивизии x M /X N = X (M — N) (xy) n = x n . Y N
Правило мощности фракции (x /y) N = x N /Y N
88
88
888
Zero Exponent (x) 0 = 1, if x ≠ 0
One Exponent (x) 1 = x
Negative Exponent x -n = 1/x n
Fractional Exponent x m/n = n √(x) m

Примечание: Если основание уравнения одинаковое, мы можем приравнять показатели.

Примеры вопросов 

Вопрос 1. Решите следующее: 

  • 2.2.2.2
  • 3 2 .
  • 2 5 /2 3  
  • [(3) 1 ]
  • 4 3/2  
  • (4/3)

Решение:

  • 2.2.2.2 = 2 4 = 16
  • 3 2 .3 3 = 3 (2 + 3) = 3 5 = 243
  • (4,5). 2 = 4 2 .5 2 = (16) 4
  • 2 5 /2 3 = 2 (5-3) = 2 2 = 4
  • [(3) 1 ] 3

    8 20007 (1.2)

    = 3 2 = 9
  • 4 3/2 = √(4) 3 = √64 = 8
  • (4/3) 90 90 2 2 2 = 16/9

Вопрос 2: Упрощение:

  1. (2 3 ÷ 2 4 ) -2 . 2) ÷ 4 2   
  2. 3 3 .4 2 /6 4   
  3. (3 -1 + 2 -2 + 4 -1 )

Решение:

  1. (2 3 ÷ 2 4 -2 7 -2 7 -2 7 -2 7 -2 7 -2

    -2

    -2
  2. . 2 3 = (2 3 /2 4 ) -2 .2 3 = [2 (3 – 4) ] -2 90 0 8 .2 39000 1 ] -2 .2 3 = 2 (-1).(-2) .2 3 = 2 2 .2 3 = 2 50008 = 32
  3. 3 (-2) ÷ 4 2 = 1/(3) 2 (4) 2 = 1/9.16 = 1/144
  4. 3 3 .4 2 2/144
  5. 3 3 . /6 4 = 3 3 .4 2 /(2.3) 4 = 3 3 .2 4 /2 4 . 3 4 = 1/3
  6. ( 3 -1 + 2 -2 + 4 -1 ) = (1/3 + 1/2 2 +1/4) = (1/3 + 1/4 + 1/4) = 5/6

Вопрос 3: Найдите значение x, если (4) x + 12 = (4) 2x + 6 . (2) 6

Решение:

(4) x + 12 = (4) 2x + 6 . 2 2 ) 3

(4) х+12 = (4) 2х+6 .(4) 3

(4) х+00 +6+3

(4) x+12 = (4) 2x+9

Так как основания равны, степени приравниваются

x +12 = 2x + 9

12x – x = 2x – 9

x = 3

Question 4: Find the value of {343 4/3 } 1/4

Solution: 

{343 4/3 } 1/4   = {(7 3 ) 4/3 } 1/4  

= {7} 3. (4/03) 9.(00/48) 9.

Вопрос 5: Найдите значение x + y, если:

(81) y = 27/(3) x , 4 y = 256

Решение:

(3 4 ) Y = (3 3 )/(3) x

(3) 4Y = x

(3) 4 0008 = = (3) 3-x

Так как основания равны, то приравниваются степени

4y = 3-x ⇢ Уравнение (1)

4 y = 256

8

7 9 0

4

y = 4

Подставляя значение y в уравнение 1,

4,4 = 3-x

16 = 3-x

x = -13

Теперь нам нужно найти значение x + y

x + y = -13+4 = -9

Вопрос 6: Если ( -9) 2x+7 = (-9) x . 81, затем найдите значение (x 2 + 1)/(x 2 — 12).

Решение:

(-9) 2x+7 = (-9) x . 81

(-9) 2x+7 = (-9) x . (-9) 2

(-9) 2x+7 = (-9) x+2

Так как основания равны, то степени приравниваются

2x + 7 = x + 2

2x – x = 2 – 7

x = -5

Теперь нам нужно найти значение  (x 2 + 1)/(x 2 – 12) 

(x 2 + 1)/(x 2 – 12) )  = [(-5) 2 + 1]/[(-5) 2 – 12]

= [25 + 1]/[25 – 12]

= 26/13  

(x 2 + 1)/(х 2 -12) = 2

Вопрос 7: Найдите мультипликативную обратную: -[(-13) -1 ] 2 ÷ (91) -1

Решение:

222229

Решение:

9

Пусть x = [(-13) -1 ] 2 ÷ (91) -1  

x = (-13) -2 ÷ (91) -1

3 = (-1 

3 1/13 2 ) ÷ (1/91)

= (-1/13 2 ) × 91

x = -7/13

Мультипликативная обратная величина определяется как 1/x, т. е.

1/x = 1/(-7/13)

1/x = -13/7

Сравнительный анализ 12 формул силы интраокулярной линзы

. 2021 Декабрь;41(12):4137-4150.

doi: 10.1007/s10792-021-01966-z. Epub 2021 27 июля.

Остин Перейра 1 , Марко М. Попович 1 , Юсуф Ахмед 2 , Джон К. Ллойд 1 3 , Шериф Эль-Дефрави 1 4 , Джон Горфинкель 1 4 , Мэтью Б. Шленкер 5 6

Принадлежности

  • 1 Департамент офтальмологии и зрительных наук, Университет Торонто, 340 College Street Unit 400, Toronto, ON, M5T 3A9, Канада.
  • 2 Медицинский факультет Университета Торонто, 1 King’s College Circle, Торонто, Онтарио, M5S 1A8, Канада.
  • 3 Центр медицинских наук Саннибрук, 2075 Bayview Avenue, Торонто, Онтарио, M4N 3M5, Канада.
  • 4 Kensington Eye Institute, 340 College Street Unit 501, Toronto, ON, M5T 3A9, Канада.
  • 5 Кафедра офтальмологии и наук о зрении, Университет Торонто, 340 College Street Unit 400, Торонто, Онтарио, M5T 3A9, Канада. [email protected].
  • 6 Kensington Eye Institute, 340 College Street Unit 501, Toronto, ON, M5T 3A9, Канада. [email protected].
  • PMID: 34318369
  • DOI: 10. 1007/с10792-021-01966-з

Остин Перейра и др. Инт офтальмол. 2021 Декабрь

. 2021 Декабрь;41(12):4137-4150.

дои: 10.1007/s10792-021-01966-з. Epub 2021 27 июля.

Авторы

Остин Перейра 1 , Марко М. Попович 1 , Юсуф Ахмед 2 , Джон К. Ллойд 1 3 , Шериф Эль-Дефрави 1 4 , Джон Горфинкель 1 4 , Мэтью Б. Шленкер 5 6

Принадлежности

  • 1 Департамент офтальмологии и зрительных наук, Университет Торонто, 340 College Street Unit 400, Toronto, ON, M5T 3A9, Канада.
  • 2 Медицинский факультет Университета Торонто, 1 King’s College Circle, Торонто, Онтарио, M5S 1A8, Канада.
  • 3 Центр медицинских наук Саннибрук, 2075 Bayview Avenue, Торонто, Онтарио, M4N 3M5, Канада.
  • 4 Kensington Eye Institute, 340 College Street Unit 501, Toronto, ON, M5T 3A9, Канада.
  • 5 Кафедра офтальмологии и наук о зрении, Университет Торонто, 340 College Street Unit 400, Торонто, Онтарио, M5T 3A9, Канада. [email protected].
  • 6 Kensington Eye Institute, 340 College Street Unit 501, Toronto, ON, M5T 3A9, Канада. [email protected].
  • PMID: 34318369
  • DOI: 10. 1007/с10792-021-01966-з

Абстрактный

Цель: Оценить точность расчета силы 12 интраокулярных линз (ИОЛ): Barrett Universal II, EVO, Haigis, Hill-RBF версии 2.0, Hoffer Q, Holladay 1, Holladay 2, Kane, Olsen, SRK/T, Super Formula и T2.

Методы: В этой ретроспективной последовательной серии случаев случаи экстракции катаракты и имплантации ИОЛ в Торонто, Канада, были набраны в период с 2017 по 2019 год.. Прогнозы рефракции сравнивали с наблюдаемым сферическим эквивалентом через 1 месяц после операции, чтобы определить ошибку рефракции для каждой когорты формулы. Анализ подгрупп разделил глаза на короткие (≤ 22,5 мм), промежуточные (22,5–25,5 мм) и длинные (≥ 25,5 мм) осевые длины (AL) когорты. Первичным результатом был процент случаев в пределах ± 0,50 дптр от ошибки рефракции.

Полученные результаты: Всего было проанализировано 764 случая катаракты. Формулы с самым высоким процентом глаз в пределах ± 0,50 дптр от ошибки рефракции в порядке убывания: Kane (77,7%), Barrett Universal II (77,4%), EVO (76,6%), T2 (76,4%), Super (75,9%).%), Holladay 1 (75,4%), Hill-RBF 2.0 (74,7%), SRK/T (72,6%), Hoffer Q (72,5%), Haigis (71,7%), Olsen (67,4%) и Holladay 2 (67,3%). %). Для глаз с коротким AL наиболее точна формула Holladay 1 (n = 69, 78,3% в пределах ± 0,50 дптр), а для глаз с длинным AL наиболее точна формула Barrett Universal II (n = 116, 76,7% в пределах ± 0,50 дптр). 0,50D). Формулы Кейна, Барретта, EVO, T2 и Super привели к значительно более низкой средней абсолютной ошибке по сравнению с расчетами с открытым исходным кодом с оптимизированными константами линзы (значение p: <0,001–0,042).

Выводы: Формула Кейна оказалась наиболее точной формулой для общего анализа. Расчет Holladay 1 был наиболее точным для коротких AL, тогда как Barrett Universal II был лучше для длинных AL.

Ключевые слова: Глубина передней камеры; Осевая длина; Формула; Интраокулярная линза; расчет мощности; Рефракционная ошибка.

© 2021. Автор(ы), по эксклюзивной лицензии Springer Nature B.V.

Похожие статьи

  • Сравнение точности 11 формул расчета оптической силы интраокулярной линзы.

    Кармона-Гонсалес Д., Кастильо-Гомес А., Паломино-Баутиста С., Ромеро-Домингес М., Гутьеррес-Морено М.А. Кармона-Гонсалес Д. и др. Eur J Офтальмол. 2021 сен; 31 (5): 2370-2376. дои: 10.1177/1120672120962030. Epub 2020, 15 октября. Eur J Офтальмол. 2021. PMID: 33054421

  • Оценка точности новых и обновленных формул расчета силы интраокулярной линзы на 10 930 глазах от Национальной службы здравоохранения Великобритании.

    Дарси К., Ганн Д., Тавассоли С., Воробей Дж., Кейн Дж.Х. Дарси К. и др. J Катаракта рефракта Surg. 2020 янв;46(1):2-7. doi: 10.1016/j.jcrs.2019.08.014. J Катаракта рефракта Surg. 2020. PMID: 32050225

  • Сравнение формулы интраокулярной линзы при осевой дальнозоркости с интраокулярной линзой большой мощности 30 и более диоптрий.

    Кейн JX, Меллес РБ. Кейн JX и др. J Катаракта рефракта Surg. 2020 сен;46(9):1236-1239. doi: 10.1097/j.jcrs.0000000000000235. J Катаракта рефракта Surg. 2020. PMID: 32384418

  • Точность формулы расчета оптической силы интраокулярной линзы: сравнение 12 формул для трифокальной гидрофильной интраокулярной линзы.

    Роча-де-Лоссада К., Колменеро-Рейна Э., Фликьер Д. , Кастро-Алонсо Ф.Х., Родригес-Ратон А., Гарсия-Мадрона Х.Л., Пераса-Ньевес Х., Санчес-Гонсалес Х.М. Роча-де-Лоссада С. и соавт. Eur J Офтальмол. 2021 ноябрь;31(6):2981-2988. дои: 10.1177/1120672120980690. Epub 2020 18 декабря. Eur J Офтальмол. 2021. PMID: 33339479

  • Точность формул расчета интраокулярной линзы у больных катарактой с крутой кривизной роговицы.

    Чжан С., Дай Г., Пазо Э.Е., Сюй Л., У С., Чжан Х., Линь Т., Хэ В. Чжан С и др. ПЛОС Один. 20 ноября 2020 г .; 15 (11): e0241630. doi: 10.1371/journal.pone.0241630. Электронная коллекция 2020. ПЛОС Один. 2020. PMID: 33216749 Бесплатная статья ЧВК.

Посмотреть все похожие статьи

использованная литература

    1. Pager CK (2004)Ожидания и результаты хирургии катаракты: проспективный тест двух моделей удовлетворенности. Арка Офтальмол 122 (12): 1788–179.2 — DOI
    1. Aristodemou P, Knox Cartwright NE, Sparrow JM, Johnston RL (2011) Выбор формулы: hoffer Q, holladay 1 или SRK/T и результаты рефракции в 8108 глазах после операции по удалению катаракты с биометрией с помощью частичной когерентной интерферометрии. J Cataract Refract Surg 37: 63–71 — DOI
    1. Сиддики А.А., Девган У. (2018) Освоение расчетов линз: новые формулы и сравнения. Отчет Curr Ophthalmol 6 (4): 233–236. — DOI
    1. Савини Г. , Ди Майта М., Хоффер К.Дж. и др. (2020) Сравнение 13 формул для расчета мощности ИОЛ с измерениями частичной когерентной интерферометрии. Бр Дж Офтальмол. https://doi.org/10.1136/bjophthalmol-2020-316193 — DOI — пабмед
    1. Тарони Л., Хоффер К.Дж., Барбони П., Скьяно-Ломориелло Д., Савини Г. (2020)Результаты расчета мощности ИОЛ с использованием измерений вращающейся шейпфлюг-камерой в сочетании с частичной когерентной интерферометрией. J Cataract Refract Surg 46 (12): 1618–1623 — DOI 90,75

      Они берут n и дают в результате e

      Как бы я изменил это, чтобы взять e и получить n . Я не очень разбираюсь в математике, это еще одна проблема.

      Я имею в виду, что я получаю для сложения просто вычитание, а для умножения просто деление, но даже тогда я не знаю, что делать, когда дело доходит до уравнения с несколькими частями, но тогда у вас также есть показатели степени и все такое смешанное. Я не знаю, с чего начать переворачивать эти формулы.

      Буду очень признателен за любую помощь или руководство, и это для личного проекта, не связанного с моей основной работой, это не связано со школой или работой, но это важно для меня, поэтому я действительно хочу это записать.

      Заранее спасибо

      • возведение в степень

      $\endgroup$

      8

      $\begingroup$

      Проблема относительно проста, и я думаю, что небольшое формальное определение решит проблему. Во-первых, обратные функции. Имея функцию $f(x)=y$, то есть функцию, в которой мы подставляем $x$, чтобы получить $y$, мы можем определить аналогичную функцию $f^{-1}(y)=x$, где мы вставляем $y$, чтобы получить $x$. {-1}(y)$, мы находим $y$. Например, если нам дано $y=x+1$, мы получим $x=y-1$. 9{-1}\left(\frac{1855-18 e}{375 \sqrt{15}}\right)\right)$$, что не представляет проблемы с карманным калькулятором.

      $\endgroup$

      $\begingroup$

      Поскольку вы не очень хорошо разбираетесь в математике, рекомендую посмотреть отличное видео Ви Харт, в котором она причудливым образом объясняет, что элементарная алгебра (включая дробные показатели) стоит всего $+1$. Например, $+3 = +1+1+1$. Сделаем это модно. Продукт представляет собой причудливый способ $+1$. Например, $12 = 3 \times 4$ означает, что $12$ равно $+3$ при подсчете в 9{\ гидроразрыва {1} {3}} $.

      Третье уравнение действительно сложнее объяснить, но, как объяснено в ответе Клода Лейбовича, вам нужно найти решение кубического уравнения в $n$.

      $\endgroup$

      Твой ответ

      Зарегистрируйтесь или войдите в систему

      Зарегистрируйтесь с помощью Google

      Зарегистрироваться через Facebook

      Зарегистрируйтесь, используя электронную почту и пароль

      Опубликовать как гость

      Электронная почта

      Требуется, но никогда не отображается

      Опубликовать как гость

      Электронная почта

      Требуется, но не отображается

      Нажимая «Опубликовать свой ответ», вы соглашаетесь с нашими условиями обслуживания, политикой конфиденциальности и политикой использования файлов cookie

      экспонентов в Excel | Как использовать экспоненты в Excel? (2 метода)

      Показатель степени в Excel — это та же экспоненциальная функция в Excel, например, в математике, где число возводится в степень или степень другого числа. Мы можем использовать показатели степени двумя способами: функция POWER на листе Excel принимает два аргумента, один как число, а другой как показатель степени, или мы можем использовать символ степени с клавиатуры.

      Содержание
      • Показатель степени в формуле Excel
        • Как использовать показатель степени в формуле Excel?
          • Метод №1 – Использование функции мощности
          • Метод №2 – Использование базовой мощности
          • Метод №3 – Использование функции EXP
          • Метод №4 – Использование текстовых показателей
        • Что следует помнить
        • Рекомендуемые статьи